June 2019 to March 2020 - Sleepy Classes

123
Economy (PRE-Mix) June 2019 to March 2020 Visit our website www.sleepyclasses.com or our YouTube channel for entire GS Course FREE of cost Also Available: Prelims Crash Course || Prelims Test Series

Transcript of June 2019 to March 2020 - Sleepy Classes

Economy (PRE-Mix)

June 2019 to March 2020

Visit our website www.sleepyclasses.com or

our YouTube channel for entire GS Course FREE of cost

Also Available: Prelims Crash Course || Prelims Test Series

1. Which of the following is/are most likely the impact(s) of a rate cut by RBI?

1. Increase in GDP Growth Rate

2. Increase in Inflation

3. Increase in Credit to Deposit Ratio

A. 1 and 2 only

B. 2 and 3 only

C. 1 and 3 only

D. All of the above

Answer: A

Explanation

• It reduces the interest rates, leading to a lesser amount of interest, bringing down the overall cost of

the loan. However, this will come into effect only if banks decide to pass on the benefit to their

customers.

• The reduction in the repo rate means that industries may be able to get loans at cheaper interest rates

from lenders. It can result in commodities becoming cheaper due to lower interest costs. It also

strengthens domestic growth impulses by spurring private investment. A rate cut can increase

liquidity in the economy and hence drive up the cost of goods and services, leading to increase in

inflation. However, Deposit Ratio is not impacted by Repo Rate as deposits are governed by Cash

Reserve Ratio (CRR) and other factors.

2. Which of the following correctly explains on-tap ‘licensing' as used by RBI?

1. Eligible entities need to wait for RBI to start the licensing process

2. It is an on-supply basis licensing process

A. 1 only

B. 2 only

C. Both are correct

D. None is correct

Answer: D

Explanation

It enables entities to approach the RBI for obtaining licences for starting banks or small finance banks on

meeting laid-down criteria. Eligible entities would not have to wait for licences as it would be available

on demand basis.

www.YouTube.com/SleepyClasses www.SleepyClasses.com

1

Jashanjot Sidhu

3. Which of the following is/are true about G20?

1. Apart from European Union, ASEAN is the only other grouping present as a member

2. While India is a member, China is not

A. 1 only

B. 2 only

C. Both are correct

D. None is correct

Answer: D

Explanation

The G20 (Or Group Of Twenty)

It is an international forum for the governments and central bank governors from 19 countries and the

European Union. The EU is represented by the European Commission and by the European Central

Bank. Membership includes:

4. Which of the following is/are true about Competition Commission of India (CCI)?

1. It ensures that monopolies are not created in the country

2. It regulates combinations such as acquisition, acquiring of control and Mergers

A. 1 only

B. 2 only

C. Both are correct

D. None is correct

Answer: B

Explanation

• A fair competition in market is essential to achieve this objective. CCI‘s goal is to create and sustain

fair competition in the economy that will provide a level playing field‘ to the producers and make the

markets work for the welfare of the consumers.

www.YouTube.com/SleepyClasses www.SleepyClasses.com

2

Jashanjot Sidhu

• The Competition Act, 2002, as amended by the Competition (Amendment) Act, 2007, prohibits

anticompetitive agreements, abuse of dominant position by enterprises and regulates

combinations (acquisition, acquiring of control and M&A), which causes or likely to cause an

appreciable adverse effect on competition within India.

• The objectives of the Act are sought to be achieved through the Competition Commission of India,

which has been established by the Central Government with effect from 14th October 2003.

• CCI consists of a Chairperson and 6 Members appointed by the Central Government.

5. Which of the following is/are correct in context of RTGS (Real Time Gross Settlement)?

1. The minimum amount to be remitted through RTGS (Real Time Gross Settlement) is ₹2,00,000.

2. RTGS is not a 24x7 system.

A. 1 only

B. 2 only

C. Both are correct

D. None is correct

Answer: C

Explanation

RTGS

• It is a system where there is continuous and real-time settlement of fund-transfers, individually on a

transaction by transaction basis (without netting).

✓ 'Real Time' means the processing of instructions at the time they are received; 'Gross Settlement'

means that the settlement of funds transfer instructions occurs individually.

• NEFT is an electronic fund transfer system in which the transactions received up to a particular time

are processed in batches. Contrary to this, in RTGS, the transactions are processed continuously on a

transaction by transaction basis throughout the RTGS business hours.

• RTGS is not a 24x7 system.

• The RTGS service window for customer transactions is available to banks from 8 am to 4:30 pm on a

working day, for settlement at the RBI end. However, the timings that the banks follow may vary from

bank to bank.

• The RTGS system is primarily meant for large value transactions. The minimum amount to be

remitted through RTGS is ₹2,00,000 with no upper or maximum ceiling.

6. Which of the following is/are correct about NITI Aayog?

1. It was formed via an act of Parliament on January 1, 2015

www.YouTube.com/SleepyClasses www.SleepyClasses.com

3

Jashanjot Sidhu
Jashanjot Sidhu
Jashanjot Sidhu
Jashanjot Sidhu
Jashanjot Sidhu
Jashanjot Sidhu

2. The Team India Hub in NITI leads the engagement of states with the Central government.

3. The Knowledge and Innovation Hub in NITI builds NITI‘s think-tank capabilities.

A. 1 and 2 only

B. 2 and 3 only

C. 1 and 3 only

D. None of the above

Answer: B

Explanation

• The National Institution for Transforming India, also called NITI Aayog, was formed via a resolution of

the Union Cabinet on January 1, 2015. NITI Aayog provides relevant technical advice to the Centre

and States.

✓The Government of India, in keeping with its reform agenda, constituted the NITI Aayog to replace

the Planning Commission instituted in 1950.

• NITI Aayog attempts to foster Cooperative Federalism. At the core of NITI Aayog‘s creation are two

hubs – Team India Hub and the Knowledge and Innovation Hub.

• The Team India Hub leads the engagement of states with the Central government. While the

Knowledge and Innovation Hub builds NITI‘s think-tank capabilities. These hubs reflect the two key

tasks of the Aayog.

• PM is the Chairperson of NITI.

7. Cabinet recently approved the New Delhi International Arbitration Centre Bill, 2019. In this regard,

consider the following statements

1. It will promote 'Ease of Doing Business' in India

2. It is being proposed based on the recommendations of a High-Level Committee (HLC), headed by

Mr. Justice B.N. Srikrishna

A. 1 only

B. 2 only

C. Both are correct

D. None is correct

Answer: C

Explanation

• In view of the provisions of the Article 107(5) and 123(2) of the Constitution, the New Delhi

International Arbitration Centre Bill, 2019 is proposed to be introduced in the Parliament which will

replace the New Delhi International Arbitration Centre Ordinance, 2019.

www.YouTube.com/SleepyClasses www.SleepyClasses.com

4

• It will resolve International and domestic commercial disputes expeditiously by Alternative Dispute

Resolution (ADR) mechanism.

• A High-Level Committee (HLC), headed by Mr. Justice B.N. Srikrishna was constituted in the year

2017.

• The HLC recommended that the Government may take over the International Centre For Alternative

Dispute Resolution (ICADR), an existing institution which has been established in the year 1995 using

the public funds and develop it as an Institution of National Importance.

• Taking into consideration the HLC's recommendations, a Bill, namely the New Delhi International

Arbitration Centre (NDIAC) Bill 2018 was approved for introduction in the Parliament by the Cabinet

in December, 2017.

New Delhi International Arbitration Centre (NDIAC)

• A hub of institutionalized arbitration and promote 'Ease of Doing Business' in India.

• In view of the provisions of the Article 107 (5) and 123 (2) of the Constitution, the New Delhi

International Arbitration Centre Bill, 2019 is proposed to be introduced in the Parliament which will

replace the New Delhi International Arbitration Centre Ordinance, 2019.

• The New Delhi International Arbitration Centre (NDIAC) will be headed by a Chairperson, who has

been a Judge of the Supreme Court or a Judge of a High Court or an eminent person, having special

knowledge and experience in the conduct or administration of arbitration, law or management, to be

appointed by the Central Government in consultation with the Chief Justice of India.

• Besides, it will also have two Full-time or Part-time Members from amongst eminent persons having

substantial knowledge and experience in institutional arbitration in both domestic and international.

• In addition, one representative of a recognized body of commerce and industry shall be nominated on

rotational basis as a Part-time Member.

• The Secretary, Department of Legal Affairs, Ministry of Law & Justice, Financial Adviser nominated by

Department of Expenditure, Ministry of Finance and Chief Executive Officer, NDIAC will be ex-officio

Members.

8. Base erosion and profit shifting relates with

1. Tax holidays given by governments to promote business activity in remote or relatively under-

developed places

2. It usually results in little or no tax being paid

A. 1 only

B. 2 only

C. Both are correct

D. None is correct

Answer: B

www.YouTube.com/SleepyClasses www.SleepyClasses.com

5

Jashanjot Sidhu
Jashanjot Sidhu
Jashanjot Sidhu
Jashanjot Sidhu
Jashanjot Sidhu
Jashanjot Sidhu

Explanation

Base erosion and profit shifting refers to tax planning strategies that exploit gaps and mismatches in tax

rules to artificially shift profits to low or no-tax locations where there is little or no economic activity,

resulting in little or no tax being paid.

India has ratified the Multilateral Convention to Implement Tax Treaty Related Measures to Prevent

Base Erosion and Profit Shifting, which was signed by previous Finance Minister at Paris in June 2017.

The Multilateral Convention is an outcome of the OECD/G20 Project to tackle Base Erosion and Profit

Shifting, also known as the BEPS Project.

9. Which of the following is/are true about economic census?

1. It is the complete count of all establishment located within the geographical boundary of India.

2. It is conducted by Ministry of Finance.

A. 1 only

B. 2 only

C. Both are correct

D. None is correct

Answer: A

Explanation

• Economic Census is the complete count of all establishment located within the geographical

boundary of India.

• The Economic Census provides disaggregated information on various operational and structural

variables of all establishments of the country.

• Economic Census also provides valuable insight into geographical spread/clusters of economic

activities, ownership pattern, persons engaged, etc. of all economic establishments in the country.

• The information collected during Economic Census are useful for socio-economic developmental

planning at state and district levels.

• It is being conducted by Ministry of Statistics and Programme Implementation (MoSPI) in 2019.

✓Six Economic Censuses, (EC) have been conducted by the Union Ministry of Statistics and Program

Implementation till date.

10.In news, double deflation is used in context of:

A. Calculating the trend of price rise in an Economy

B. Estimating real value added of an industry

C. Calculating the trend of exports/imports in an Economy www.YouTube.com/SleepyClasses

www.SleepyClasses.com 6

Jashanjot Sidhu
Jashanjot Sidhu
Jashanjot Sidhu
Jashanjot Sidhu
Jashanjot Sidhu

D. Estimating the optimum amount of Foreign Exchange required by a country

Answer: B

Explanation

• Double Deflation is in news because of controversies surrounding GDP growth rate.

• It is the technique used to estimate real value added of an industry. In the double deflation method,

real value added is measured as the difference between real gross output and real intermediate

inputs.

• Double deflation is the conceptually preferred method of computing real value added because it

requires fewer assumptions about the relationships among gross output and intermediate inputs.

• India has subscribed to the Special Data Dissemination Standard (SDDS) of the International

Monetary Fund (IMF).

• The IMF had raised certain issues on the usage of double deflation in the Indian GDP series and India

had informed IMF that the existing data availability does not permit its application in India at present.

In view of divergent views, the Advisory Committee on National Accounts Statistics (ACNAS) had not

agreed to adoption of double deflation at this stage. Moreover, double deflation is used in only a few

countries that have a Producers Price Index (PPI) to deflate the inputs.

11.Consider the following statements regarding Kimberley process.

1. India is hosting this year‘s inter-sessional meeting of Kimberley Process in Delhi

2. Kimberley is a city situated in South Africa

3. Kimberley Process is open to all countries

Which of these options is/are incorrect?

A. 3 only

B. 2 and 3 only

C. 1 only

D. None of the above

Answer: C

Explanation

• India is hosting this year‘s inter-sessional meeting of Kimberley Process in Mumbai.

• Kimberley is a city situated in South Africa and the process is open to all countries.

12.Which of the statements is true in the context of WTO?

1. Kazakhstan is organizing the Twelfth Ministerial Conference (MC12) to be held in 2020.

www.YouTube.com/SleepyClasses www.SleepyClasses.com

7

Jashanjot Sidhu
Jashanjot Sidhu
Jashanjot Sidhu
Jashanjot Sidhu
Jashanjot Sidhu
Jashanjot Sidhu
Jashanjot Sidhu
Jashanjot Sidhu

2. The previous Ministerial Conference (MC11) was held in Geneva in December 2017.

A. 1 only

B. 2 only

C. Both are correct

D. None is correct

Answer: A

Explanation

• Kazakhstan is organizing the Twelfth Ministerial Conference (MC12) to be held in 2020.

• The previous Ministerial Conference (MC11) was held in Buenos Aires in December 2017.

13.Which of the following statements are true about Kaladan multimodal transit transport project?

1. India, Myanmar and Bangladesh are involved in this project.

2. Sea and Railways are the only modes of transport proposed under this project

A. 1 only

B. 2 only

C. Both are correct

D. None is correct

Answer: D

Explanation

India & Myanmar are involved in this project. It involves all the modes of transport that is road rail

waterways and sea.

14.Which of the following statements is incorrect:

A. The APEDA was established by the Government of India under the Agricultural and Processed Food

Products Export Development Authority Act passed by the by the Parliament in December 1985.

B. APEDA has been entrusted with the responsibility to monitor import of sugar

C. APEDA comes under the ambit of Ministry of Agriculture

D. APEDA replaced the Processed Food Export Promotion Council (PFEPC)

Answer: C

Explanation

The Agricultural and Processed Food Products Export Development Authority (APEDA) comes under

the ambit of Ministry of Commerce and Industry www.YouTube.com/SleepyClasses

www.SleepyClasses.com 8

Jashanjot Sidhu
Jashanjot Sidhu
Jashanjot Sidhu

APEDA was established by the Government of India under the Agricultural and Processed Food

Products Export Development Authority Act passed by the Parliament in December, 1985.

APEDA is mandated with the responsibility of export promotion and development of the following

scheduled products:

• Fruits, Vegetables and their Products.

• Meat and Meat Products.

• Poultry and Poultry Products.

• Dairy Products.

• Confectionery, Biscuits and Bakery Products.

• Honey, Jaggery and Sugar Products.

• Cocoa and its products, chocolates of all kinds.

• Alcoholic and Non-Alcoholic Beverages.

• Cereal and Cereal Products.

• Groundnuts, Peanuts and Walnuts.

• Pickles, Papads and Chutneys.

• Guar Gum.

• Floriculture and Floriculture Products.

• Herbal and Medicinal Plants. In addition to this, APEDA has been entrusted with the responsibility to

monitor import of sugar.

15.Which of the following given statements is/are correct?

1. The Shanghai Five mechanism created in 1996 had China, Kazakhstan, Kyrgyzstan, Russia and

Tajikistan as its members.

2. The declaration of Shanghai Cooperation Organisation was signed in 2001 with Uzbekistan as its

member.

3. India and Pakistan became SCO’s full members in the year 2017.

A. 2 only

B. 1 and 2 only

C. 1 and 3 only

D. All of the above

Answer: D

www.YouTube.com/SleepyClasses www.SleepyClasses.com

9

Jashanjot Sidhu
Jashanjot Sidhu
Jashanjot Sidhu
Jashanjot Sidhu
Jashanjot Sidhu
Jashanjot Sidhu
Jashanjot Sidhu
Jashanjot Sidhu

Explanation

• SCO Founded in 2001 in Shanghai 8 members: China, Kazakhstan, Kyrgyzstan, Russia, Tajikistan,

Uzbekistan, India and Pakistan

• Started as Shanghai Five (First Five mentioned above) HQ : Beijing, China

• 2015 - SCO decided to admit India and Pakistan as full members They joined as full members in 2017

in Astana, Kazakhstan, June 2017

• Main activities Cooperation on security, military activities, and economic and cultural cooperation,

Regional Antiterrorism Structure (RATS)

• Latest Summit: Bishkek, Kyrgyzstan

16.Consider the following straits and regions:

Straits

1. Strait of Malacca

2. Strait of Bab-El-Mandab

3. Strait of Tartary

4. Starit of Hormuz

Regions

South East Asia

West Africa

East Asia

West Asia

Which of these is/are incorrectly matched?

A. 2 only

B. 3 and 4 only

C. 1, 2 and 3 only

D. All are correctly matched

Answer: A

Explanation

www.YouTube.com/SleepyClasses www.SleepyClasses.com

10

Jashanjot Sidhu

17.The components used to make stainless steel include:

1. Chromium

2. Silicon

3. Carbon

4. Nickel

5. Molybdenum

6. Aluminium

Which of the following is correct?

A. 1, 3 and 6 only

B. 2, 3, 4 and 6 only

C. 2, 3 and 5 only

D. All of the above ingredients

Answer: D

18.Which of the following statements is correct in the context of India and renewable energy

A. India plans to establish a Renewable Energy capacity of 500 GW by 2022

B. India targets to install 175 GW of Solar Energy capacity by 2022

C. India‘s renewable energy capacity target by 2022 also includes 60 GW from wind, 10 GW from bio-

power and 5 GW from small hydro-power

D. International Solar Alliance (ISA) has been initiated by G-20 Countries

Answer: C

Explanation

• India targets to install 175 GW of renewable energy capacity by 2022. The 175 GW of renewable

energy capacity target by 2022 includes 100 GW from solar, 60 GW from wind, 10 GW from bio-

power and 5 GW from small hydro-power.

www.YouTube.com/SleepyClasses www.SleepyClasses.com

11

Jashanjot Sidhu
Jashanjot Sidhu

• The International Solar Alliance is an alliance of more than 122 countries initiated by India, most of

them being sunshine countries, which lie either completely or partly between the Tropic of Cancer

and the Tropic of Capricorn.

19.Which of the following correctly describes the section 7 of the RBI Act:

A. It empowers the RBI to be the banker to the Central Government

B. It empowers the Government to issue directions to RBI

C. It decides the composition of central board of Reserve Bank of India

D. It gives authority to RBI to issue bank notes

Answer: B

Explanation

RBI Act And Various Sections

• Empowers the Government to issue directions to RBI - Section 7

• Composition of central board of Reserve Bank of India - Section 8

• Being banker to the Central Government - Section 21

• Right to issue bank notes - Section 22

20.Paris-Pact Initiative is related with which of the UN bodies?

A. UNEP

B. UNDP

C. UNODC

D. UNCTAD

Answer: C

21.Which of the following statements is correct in context of CPI?

1. The Housing Group has the maximum weightage in CPI-Rural

2. Food and Beverages Group has the maximum weightage in CPI-Urban and CPI-Combined

A. 1 only

B. 2 only

C. Both are correct

D. None is correct

Answer: B

www.YouTube.com/SleepyClasses www.SleepyClasses.com

12

Jashanjot Sidhu
Jashanjot Sidhu
Jashanjot Sidhu
Jashanjot Sidhu
Jashanjot Sidhu
Jashanjot Sidhu
Jashanjot Sidhu

Explanation

• Housing Group is not the part of CPI-Rural

• The highest weightage in CPI-Rural is of Food and Beverages

22.Consider the following statement(s) about S400 Triumf.

1. It can shoot down hostile aircraft, drone and hypersonic targets.

2. Turkey has decided to forego the S400 air defence system under the US pressure.

Which of the above statements are correct?

A. 1 only

B. 2 only

C. Both 1 and 2

D. Neither 1 nor 2

Answer: A

23.Consider the following statement(s) in the context of Non-permanent members of UNSC.

1. India has only been elected as a non permanent member once since the creation of UNSC.

2. General Assembly elects five non-permanent every year for two-year a term.

Which of the above statements are correct?

A. 1 only

www.YouTube.com/SleepyClasses www.SleepyClasses.com

13

Jashanjot Sidhu

B. 2 only

C. Both 1 and 2

D. Neither 1 nor 2

Answer: B

24.Consider the following statements in the context of sovereign gold bonds

1. The bond can be purchased from both NSE (National Stock Exchange) and BSE (Bombay Stock

Exchange).

2. The bonds will have a sovereign guarantee as they will be issued on behalf of the Government of

India by the RBI.

3. The bonds will be available both in demat and paper form.

4. Bond can be used as collateral for loans.

Which of the above given statements is/are correct?

A. 3 only

B. 2, 3 and 4 only

C. 2 and 4 only

D. All are correct

Answer: D

25.Consider the following statements about MCLR

1. MCLR is the maximum interest rate of a bank above which it cannot lend.

2. It has replaced the base rate system.

3. It is revised annually by keeping factors like repo rate under consideration.

Which of these is/are incorrect?

A. 2 and 3 only

B. 1 and 3 only

C. All are incorrect

D. All are correct

Answer: B

Explanation

• It is the minimum interest rate of bank below which it cannot lend.

www.YouTube.com/SleepyClasses www.SleepyClasses.com

14

Jashanjot Sidhu
Jashanjot Sidhu
Jashanjot Sidhu
Jashanjot Sidhu
Jashanjot Sidhu

• There is no such rule of annual revision. It can be revised by Bank depending on the circumstances

26.The term differential voting rights (DVR), often seen in news, is used in context of

A. The shares that give special rights to shareholders over dividends ahead of equity shareholder.

B. Shares that provide lesser voting rights but higher dividends.

C. Shares that provide higher voting rights and higher dividends.

D. None is correct

Answer: B

Explanation

• A DVR share is like an ordinary equity share, but it provides fewer voting rights to the shareholder.

DVR shares are priced lower at issuance and offer higher dividends in return of limited voting

rights.

• Companies issue DVR shares for prevention of a hostile takeover and dilution of voting rights. It also

helps strategic investors who do not want control, but are looking at a reasonably big investment in

a company.

• At times, companies issue DVR shares to fund new large projects, due to fewer voting rights, even a

big issue does not trigger an open offer.

• The Companies Act permits a company to issue DVR shares when, among a few conditions are met.

Recently, SEBI has approved a framework for issue of differential voting rights (DVR) shares, clarified

the meaning of a few words and revised the risk management framework for mutual funds.

• The shares that give special rights to shareholders over dividends ahead of equity shareholder are

called Preference shares.

27.Consider the following statements

1. Sweden is the world leader in the number of electric vehicles (EVs) on the road.

2. NITI Aayog has proposed that after 2025, only electric vehicles will be sold.

Which of the above given statements is/are correct?

A. 1 only

B. 2 only

C. Both

D. None

Answer: D

www.YouTube.com/SleepyClasses www.SleepyClasses.com

15

Jashanjot Sidhu
Jashanjot Sidhu
Jashanjot Sidhu
Jashanjot Sidhu
Jashanjot Sidhu

Explanation

• Norway is the world leader in the number of electric vehicles on the road.

• NITI Aayog has proposed that after 2030, only electric vehicles will be sold

28.Which of the following are the components of forex?

1. Foreign Currency Assets (FCAs)

2. Special Drawing Rights with IMF

3. Gold Reserves

4. RBI’s reserve position with the IMF

Which of the above given statements is/are correct

A. 1 and 2 only

B. 2 only

C. 3 and 4 only

D. All are correct

Answer: D

29.Consider the following statements regarding Fiscal Deficit

1. Fiscal Deficit is the difference between total government expenditure and total government

receipts minus the borrowings.

2. Fiscal deficit includes the interest amount where as primary deficit excludes interest payment

amount.

3. A deficit is usually financed through borrowing from either the central bank of the country or

raising money from capital markets by issuing different instruments like treasury bills and bonds.

www.YouTube.com/SleepyClasses www.SleepyClasses.com

16

Jashanjot Sidhu

Which of the above given statements is/are correct?

A. 1 only

B. 1 and 3 only

C. 1 and 4 only

D. All are correct

Answer: D

Explanation

• Fiscal deficit is defined as the excess of total budget expenditure over total budget receipts

excluding borrowings during a fiscal year.

• It is an indication of the total borrowings needed by the government.

Fiscal deficit = Total expenditure — Total receipts excluding borrowings

• A deficit is usually financed through borrowing from either the central bank of the country or raising

money from capital markets by issuing different instruments like treasury bills and bonds.

Primary deficit = Fiscal deficit — Interest payments

• Primary deficit is defined as fiscal deficit of current year minus interest payments on previous

borrowings.

• In other words whereas fiscal deficit indicates borrowing requirement inclusive of interest payment,

primary deficit indicates borrowing requirement exclusive of interest payment (i.e., amount of loan).

30.Which of the following is/are the components of Current Account

1. Goods trade in goods

2. Services (Invisible) trade in services e.g. tourism

3. Income investment income

4. Current Unilateral Transfers donations, gifts, grants, remittances

A. 1 only

B. 1 and 3 only

C. 1, 2 and 4 only

D. 1, 2, 3 and 4 only

Answer: D

Explanation

• A current account reflects the net results of a country’s recurring financial transactions with the

rest of the world. The usual components are

www.YouTube.com/SleepyClasses www.SleepyClasses.com

17

Jashanjot Sidhu
Jashanjot Sidhu
Jashanjot Sidhu

✓Trade in goods and services

✓Net income from foreign investments

✓Direct money transfers

• Trade includes both export and import of physical goods and services.

• Income from foreign investments includes dividends and interest.

• Direct transfer is all money remitted to a home country by its citizens working abroad.

• If the net value (credit minus debit) of all these components is positive, then its current account is said

to be in surplus.

• A negative value indicates that the country runs a deficit (CAD).

• Trade in goods and services is the biggest component in the current account

31.Consider the following statements regarding Ease of Doing Business Report, 2019

1. A nation's ranking on the index is based on the average of 30 sub indices.

2. It is the second consecutive year for which India has been recognized as one of the top improvers.

3. India is the first BRICS and South Asian country to be recognized as top improvers in consecutive

years.

Which of the above statements is/are correct?

A. 1 only

B. 2 only

C. 1 and 2 only

D. 2 and 3 only

Answer: D

Explanation

• The ease of doing business index is an index created by Simeon Djankov at the World Bank Group. A

nation's ranking on the index is based on the average of 10 sub indices.

• India has recorded a jump of 23 positions against its rank of 100 in 2017 to be placed now at 77th

rank among 190 countries assessed by the World Bank.

• India's leap of 23 ranks in the Ease of Doing Business ranking is significant considering that last year

India had improved its rank by 30 places, a rare feat for any large and diverse country of the size of

India.

• As a result of continued efforts by the Government, India has improved its rank by 53 positions in last

two years and 65 positions in last four years.

• The important features of India's performance this year are

www.YouTube.com/SleepyClasses www.SleepyClasses.com

18

Jashanjot Sidhu
Jashanjot Sidhu
Jashanjot Sidhu
Jashanjot Sidhu
Jashanjot Sidhu
Jashanjot Sidhu

✓ The World Bank has recognized India as one of the top improvers for the year.

✓ This is the second consecutive year for which India has been recognized as one of the top

improvers.

• India is the first BRICS and South Asian country to be recognized as top improvers in consecutive

years.

• India has recorded the highest improvement in two years by any large country since 2011 in the

Doing business assessment by improving its rank by 53 positions. As a result of continued

performance, India is now placed at first position among South Asian countries as against 6th in 2014.

32.Consider the following statements with respect to Competition Act, 2002.

1. It has replaced The Monopolies and Restrictive Trade Practices Act, 1969.

2. It was based on the Raghavan Committee.

Which of the above statements are incorrect?

A. 1 only

B. 2 only

C. Both 1 and 2

D. Neither 1 nor 2

Answer: D

Explanation

• The Competition Act, 2002 was enacted by the Parliament of India and governs Indian competition

law. It replaced the archaic The Monopolies and Restrictive Trade Practices Act, 1969.

• The Raghavan Committee that formulated the need for a proactive competition law in lieu of the

Monopolies and Restrictive Trade Practices Act (MRTP), 1969, is of 1999 vintage.

33.Which statement is incorrect in the context of FIDC (Finance Industry Development Council)?

A. It is a self regulatory body for asset financing NBFCs

B. On of the positions from either Chairman or Co-Chairman will at all times be from small NBFCs.

C. Membership is open to all NBFCs

D. All are correct

Answer: C

Explanation

• FIDC is a self regulatory body cum Representative Body of the Non-Banking Finance Companies

(NBFCs), registered with the Reserve Bank of India.

www.YouTube.com/SleepyClasses www.SleepyClasses.com

19

Jashanjot Sidhu
Jashanjot Sidhu
Jashanjot Sidhu
Jashanjot Sidhu
Jashanjot Sidhu
Jashanjot Sidhu

• FIDC was formed in 2004.

• The Managing Committee Members of the FIDC are and will always be nominated in the manner that

there is proper representation given to small companies. Even, one of the position from either

Chairman or Co-Chairman will at all times be from small NBFCs.

• Membership is open for only those NBFCs which are registered with RBI

• FIDC will work in totally democratic and decentralized manner with full delegation of all powers and

authorities to regional chapters and affiliated associations.

• Mission

✓ To protect the interest of its members.

✓ To promote Asset Finance Business.

✓ To represent to Govt., RBI & various statutory & Trade Bodies.

✓ To promote brotherhood amongst members.

✓ To ensure fair and ethical practices among its members

34.Recently a project named Hand Made in India (HMI) was initiated by Entrepreneurship Development

Institute of India (EDII). Which of the following is true in this regard?

1. For the first phase, it is focusing entirely on Jammu & Kashmir’s artisans.

2. Entrepreneurship Development Institute of India (EDII) is a Government of India’s institute formed

in 2015 under the Ministry of Skill Development.

A. 1 only

B. 2 only

C. Both are correct

D. None is correct

Answer: D

Explanation

• Hand Made in India (HMI) project will be implemented in three years. In the first phase, it has been

extended to six centres. These handloom clusters include:

✓ Bhuj and Surendra nagar in Gujarat

✓ Bargarh in Odisha

✓ Kamrup in Assam

✓ Maheshwar in Madhya Pradesh

✓ Salem in Tamil Nadu

www.YouTube.com/SleepyClasses www.SleepyClasses.com

20

Jashanjot Sidhu
Jashanjot Sidhu
Jashanjot Sidhu
Jashanjot Sidhu
Jashanjot Sidhu
Jashanjot Sidhu
Jashanjot Sidhu
Jashanjot Sidhu
Jashanjot Sidhu
Jashanjot Sidhu

• Entrepreneurship Development Institute of India (EDII) will attempt to develop entrepreneurial

competencies in about 5,000 weavers, artisans and traders to promote hand-made art.

• Women empowerment will be given special attention in the weaving and allied activities.

Entrepreneurship Development Institute Of India (EDII)

• It is an autonomous and not-for-profit institute, set up in 1983, is sponsored by apex financial

institutions:

✓ The IDBI Bank Ltd

✓ IFCI Ltd

✓ ICICI Bank Ltd

✓ The State Bank of India (SBI)

• It is located in Gandhinagar, Gujarat.

• The Ministry of External Affairs, Govt. of India assigned to EDII the task of setting up

Entrepreneurship Development Centers in Cambodia, Lao PDR, Myanmar and Vietnam and

Uzbekistan.

• Five such centres in African region will be established very soon.

35.Often in news, interim resolution professionals are related with

A. Proceedings under Domestic Violence Act

B. Proceedings under Financial Resolution and Deposit Insurance Code

C. Proceedings under Insolvency and Bankruptcy Code

D. Proceedings under Reserve Bank Of India Act, 1934

Answer: C

Explanation

• Section 16(1) of the Insolvency and Bankruptcy Code, 2016 requires Adjudicating Authority to

appoint an interim resolution professional (IRP) within fourteen days from the insolvency

commencement date.

• As the name suggests Interim resolution professional is appointed for a very short period not

exceeding 30 days from the date of his appointment

• Insolvency and bankruptcy code 2016 was passed in May 2016 to cater to ever increasing bad loan

cases in India.

• The insolvency and bankruptcy code 2016 have a provision to appoint an interim resolution

professional. The so appointed interim resolution professional shall take care of end to end

disciplinary proceedings of the case.

www.YouTube.com/SleepyClasses www.SleepyClasses.com

21

Jashanjot Sidhu
Jashanjot Sidhu
Jashanjot Sidhu
Jashanjot Sidhu
Jashanjot Sidhu
Jashanjot Sidhu

36.Which of the following refineries are correctly matched with the place they are located at:

1. Manali Refinery Himachal Pradesh

2. Bina Refinery Rajasthan

3. Barauni Refinery Madhya Pradesh

Which of the above given statements is/are incorrect?

A. 1 only

B. 2 and 3 only

C. All of the above

D. None of the above

Answer: C

Explanation

1. Manali Refinery Tamil Nadu

2. Bina Refinery Madhya Pradesh

3. Barauni Refinery Bihar

37.Consider the following statements in context of Deendayal port trust:

1. It is the biggest non-major port in India in terms of volume handled.

2. Earlier it was known by the name of Nhava-Sheva port trust.

Which of the above given statements is/are correct?

A. 1 only

B. 2 only

C. Both 1 and 2

D. Neither 1 nor 2

Answer: D

Explanation

• Kandla, also known as the Kandla Port Trust or Deendayal Port Trust is a seaport in Kutch District of

Gujarat state in western India.

• Located on the Gulf of Kutch, it is one of major ports on west coast.

• Kandla was promoted in the 1950s as the chief seaport serving western India, after the partition of

India and Pakistan left the port of Karachi in Pakistan.

• It is the largest port of India by volume of cargo handled.

www.YouTube.com/SleepyClasses www.SleepyClasses.com

22

Jashanjot Sidhu

• Kandla Port was renamed as Deendayal Port under the India port act of 1908. Major imports

entering the Port of Kandla are petroleum, chemicals, and iron and steel and iron machinery, but it

also handles salt, textiles, and grain.

• Jawaharlal Nehru Port is also known as Nhava-Sheva Port. It is the largest container port in India.

38.Which of the following is/are true in context of PM-KISAN scheme?

1. PM KISAN is a Central Sector scheme with 100% funding from Government of India.

2. Under the scheme an income support of Rs.6000 per year in four equal instalments will be

provided to small and marginal farmer families having combined land holding/ownership of upto 2

hectares.

3. Central Governments Ministry of Agriculture will identify the farmer families which are eligible for

support as per scheme guidelines.

Which of the above given statements is/are correct?

A. 2 only

B. 1 and 2 only

C. All are correct

D. 1 only

Answer: D

Explanation

• PM KISAN is a Central Sector scheme with 100% funding from Government of India

• It has become operational from 1.12.2018.

• Under the scheme an income support of Rs.6000 per year in three equal instalments will be provided

to small and marginal farmer families having combined land holding/ownership of upto 2 hectares

• Definition of family for the scheme is husband, wife and minor children.

• State Government and UT Administration will identify the farmer families which are eligible for

support as per scheme guidelines.

• The fund will be directly transferred to the bank accounts of the beneficiaries. The first instalment for

the period 1.12.2018 to 31.03.2019 is to be provided in this financial year itself.

• There are various Exclusion Categories for the scheme.

39.Which of the following will not enable a bank to reduce the interest rate charged to the customers?

1. High rates of interest on Fixed Deposits (FD)

2. High rates of interest on Small Savings

www.YouTube.com/SleepyClasses www.SleepyClasses.com

23

Jashanjot Sidhu
Jashanjot Sidhu
Jashanjot Sidhu

3. High availability/liquidity of funds with banks

Which of the above given statements is/are incorrect?

A. 3 only

B. 1 and 2 only

C. All are correct

D. None is correct

Answer: B

Explanation

40.Which of the following is/are true in context of characteristics of a virtual currency:

1. It is a medium of exchange

2. It is a unit of account

3. It is a store of value

4. It generally has legal tender status

A. All except 4

B. 1 and 2 only

C. 3 only

D. All are correct

Answer: A

www.YouTube.com/SleepyClasses www.SleepyClasses.com

24

Explanation

Why in News

Because of Bill on Private Crypto-Currencies

• The draft bill, Banning of Cryptocurrency & Regulation of Official Digital Currency Bill, 2019,

proposes stringent fines of up to Rs 25 crore and jail terms of up to 10 years for anyone who, directly

or indirectly mines, generates, holds, sells, deals in, transfers, disposes of or issues cryptocurrency.

• There is no underlying intrinsic value of private cryptocurrencies.

• They lack the attributes of a currency and have no fixed nominal value—neither act as any store of

value nor are they a medium of exchange.

41.Impact investing deals with

A. Understanding how investments impact the investors

B. Understanding how investments can be made socially and financially impactable

C. Understanding how investments impact the governments

D. None of the above

Answer: B

Explanation

• Impact investment is a pathway to achieve the seemingly divergent goals of financial returns and

social good.

• Energy and microfinance continue to command the attention of funders the world over, as per the

Global Impact Investing Network (GIIN).

• In India, it is moving towards agriculture and healthcare in the country.

• Impact investment is being viewed as a potential game changer to address some of the world’s most

complex problems that governments alone cannot handle because of budgetary constraints.

• The promise of returns on each investment draws funders who want to do public good. In the Union

Budget, the government proposed a social stock exchange under SEBI to list social enterprises and

voluntary organisations so they can raise capital.

www.YouTube.com/SleepyClasses www.SleepyClasses.com

25

Jashanjot Sidhu
Jashanjot Sidhu

www.YouTube.com/SleepyClasses www.SleepyClasses.com

26

42.Which of the following is/are true in context of demographic dividend

1. It is referred to as the growth potential that results from shifts in a population’s age structure.

2. This transition happens largely because of a decrease in the total fertility rate.

3. India has been said to enter it in 2008.

A. 1 only

B. 2 only

C. Both 1 and 2

D. Neither 1 nor 2

Answer: C

Explanation

• United Nations Population Fund (UNFPA) defines it as the growth potential that results from shifts

in a population’s age structure. This transition happens largely because of a decrease in the total

fertility rate (TFR, which is the number of births per woman) after the increase in life expectancy gets

stabilised.

• Since 2018, India’s working-age population (people between 15 and 64 years of age) has grown

larger than the dependant population — children aged 14 or below as well as people above 65 years

of age. This bulge in the working-age population is going to last till 2055, or 37 years from its

beginning.

• Japan was among the first major economies to experience rapid growth because of changing

population structure. China entered this stage in 1994. This change in population structure alone

cannot push growth. In the late 20th century demographic dividend in Asia resulted in a seven-fold

increase in the GDP of many countries.

• In Latin America the growth was only two-fold. UNFPA points out that countries can only harness the

economic potential of the demographic dividend if they are able to provide good health, quality

education and decent employment to its entire population.

www.YouTube.com/SleepyClasses www.SleepyClasses.com

27

Jashanjot Sidhu
Jashanjot Sidhu
Jashanjot Sidhu

43.This year’s (2019) Global Innovation Index (GII) is launched globally in New Delhi. Which of the

following is the theme of this year’s global innovation index (GII):

A. Creating Smart Cities - The Future of Urban Innovation

B. Creating Efficient Employees - The Future of Labour and Artificial Intelligence

C. Creating Healthy Lives - The Future of Medical Innovation

D. Creating Clearer Skies - The Future of Innovation in Emission Control

Answer: C

Explanation

• GII theme of this year is Creating Healthy Lives - The Future of Medical Innovation. Government of

India is focusing on not just curative but preventive healthcare where wellness becomes a part of

society.

• The GII rankings are published every year by Cornell University, INSEAD and the UN World

Intellectual Property Organization (WIPO) and GII Knowledge Partners. This is the 12th edition of

the GII rankings of 129 economies based on 80 indicators ranging from intellectual property filing

rates to mobile-application creation, education spending and scientific and technical publications.

• India jumped five places to improve its position from 57th last year to 52nd in 2019. Switzerland remains number one in the GII index.

44.In news, Dalwai committee was formulated to recommend on:

A. Electric Vehicles in India

B. Doubling Farmers‘ Income in India

C. Negotiating terms with RCEP

D. Negotiating Power Purchase Agreements

Answer: B

Explanation

• The government has asked the Ashok Dalwai committee to oversee the implementation of its

recommendations so that the promise of doubling farmer’s income is achieved by 2022.

• Ashok Dalwai is the CEO of Rainfed Area Authority.

• The committee will act as an empowered body to coordinate with different ministries as well as

among various departments within the agriculture ministry.

45.Which of the following is/are true in context of Small Farmers’ Agri-Business Consortium:

1. It is chaired by Union Minister for Agriculture and Farmers Welfare

2. It is entrusted with the task of implementation of the Delhi Kisan Mandi

3. It is entrusted with the task of implementation of the National Agriculture Market Scheme www.YouTube.com/SleepyClasses

www.SleepyClasses.com 28

Jashanjot Sidhu
Jashanjot Sidhu
Jashanjot Sidhu
Jashanjot Sidhu
Jashanjot Sidhu
Jashanjot Sidhu

Which of the following is/are correct in this context?

A. 1 and 2 only

B. 3 only

C. All are correct

D. None is correct

Answer: C

Explanation

• Small Farmers Agribusiness Consortium (SFAC) is an Autonomous Society promoted by Ministry of

Agriculture, Cooperation and Farmers’ Welfare, Government of India.

• The Society is governed by Board of Management which is chaired, ex-officio, by Union Minister for

Agriculture and Farmers Welfare as the President and the Secretary, Department of Agriculture,

Cooperation and Farmers Welfare, Government of India, is the ex-officio Vice- President.

• SFAC is implementing the central schemes of Government of India namely VCA, EGCGS for

economic inclusion of small and marginal farmers in agribusiness activities.

• Society is pioneer in organising small and marginal farmers as Farmers Interest Groups, Farmers

Producers Organisation and Farmers Producers Company for endowing them with bargaining power

and economies of scale.

• Recently the Society has been entrusted with the task of implementation of the critically important

Delhi Kisan Mandi and National Agriculture Market Scheme on e-platform to progressively free

agricultural trade and offer price discovery to farmers.

46.Pradhan Mantri Shram Yogi Maan Dhan scheme is being run under the aegis of:

A. Ministry of Labour

B. Ministry of Finance

C. Ministry of External Affairs

D. Ministry of Commerce

Answer: A

Explanation

• The Government proposes to launch a mega pension yojana namely 'Pradhan Mantri Shram-Yogi

Maandhan' for the unorganised sector workers with monthly income upto Rs. 15,000.

• This pension yojana shall provide them an assured monthly pension of Rs. 3,000 from the age of 60

years on a monthly contribution. An unorganised sector worker joining pension yojana at the age of

29 years will have to contribute only Rs. 100 per month till the age of 60 years. A worker joining the

pension yojana at 18 years, will have to contribute as little as Rs. 55 per month only.

• The Government will deposit equal matching share in the pension account of the worker every month. www.YouTube.com/SleepyClasses

www.SleepyClasses.com 29

Jashanjot Sidhu
Jashanjot Sidhu
Jashanjot Sidhu
Jashanjot Sidhu
Jashanjot Sidhu
Jashanjot Sidhu
Jashanjot Sidhu
Jashanjot Sidhu

47.The code on wages bill, 2019, seeks to amend and consolidate which of the following laws:

1. Minimum Wages Act

2. Payment of Wages Act

3. Payment of Bonus Act

4. Equal Remuneration Act

5. Sharecropper Remuneration Act

6. Agricultural Labour Act

Which of the following is/are correct in this context?

A. All except 5

B. All except 5 & 6

C. 5 & 6 only

D. All of the above

Answer: B

Explanation

The Code on Wages Bill, 2019, which seeks to amend and consolidate the laws relating to wages and

bonus and matters connected therewith, will benefit 50 crore workers. The Bill subsumes four labour

laws

1. Minimum Wages Act

2. Payment of Wages Act

3. Payment of Bonus Act

4. Equal Remuneration Act

After enactment of the Code, all these four Acts would be repealed

48.Which of the following is true in context of Faster Adoption and Manufacturing of Electric vehicles in

India phase II (Fame India Phase II):

1. It has a total outlay of Rs.1000 crores over the period of three years.

2. While it encourages Electric and hybrid vehicle, it does so by emphasising on electrification of the

public transportation

Which of the following is/are correct in this context?

A. 1 only

B. 2 only

C. Both are correct

www.YouTube.com/SleepyClasses www.SleepyClasses.com

30

Jashanjot Sidhu

D. None is correct

Answer: B

Explanation

• Faster Adoption and Manufacturing of Electric Vehicles in India Phase II (FAME India Phase II)' for

promotion of Electric Mobility in the country scheme with total outlay of Rs 10000 Crores over the

period of three years will be implemented with effect from 1st April 2019.

• The main objective of the scheme is to encourage Faster adoption of Electric and hybrid vehicle by

way of offering upfront Incentive on purchase of Electric vehicles and also by way of establishing a

necessary charging Infrastructure for electric vehicles.

• Emphasis is on electrification of the public transportation that includes shared transport.

• In 3-Wheeler and 4-Wheeler segment incentives will be applicable mainly to vehicles used for public

transport or registered for commercial purposes.

• While in the e-2-Wheelers segment, the focus will be on the private vehicles

49.Commonly used in the parlance of WTO, de-minimis stands for

A. The minimal level of trade that a country can do without affecting the trade of neighbouring

exporter nations.

B. The minimal level of trade that a country can do without affecting the value of currency of

neighbouring exporter nations.

C. Maximal amounts of domestic support that are allowed even though they distort trade.

D. Minimal amounts of domestic support that are allowed even though they distort trade.

Answer: D

Explanation

• De Minimis is the minimal amounts of domestic support that are allowed even though they distort

trade.

• It is up to 5% of the value of production foe developed countries & 10% for developing.

50.Arrange the following in decreasing order of their weights in core sector:

1. Electricity

2. Crude Oil

3. Coal

4. Steel

Which of the following is/are correct in this context?

www.YouTube.com/SleepyClasses www.SleepyClasses.com

31

Jashanjot Sidhu
Jashanjot Sidhu
Jashanjot Sidhu
Jashanjot Sidhu
Jashanjot Sidhu

A. 1-2-3-4

B. 4-3-2-1

C. 1-4-3-2

D. 1-4-2-3

Answer: C

Explanation

Core Sector Slowed Down

• The eight core industries — coal, crude oil, natural gas, refinery products, fertilisers, steel, cement and

electricity slowed sharply in June, at just 0.2 per cent.

51.Recently, the regulation of housing finance companies (HFCS) has been handed over to

A. Reserve Bank of India (RBI)

B. National Housing Bank (NHB)

C. Securities and Exchange Board of India (SEBI)

D. Ministry of Finance

Answer: A

Explanation

• The RBI operationalised a Budget proposal to shift regulation of HFCs from the National Housing

Bank (NHB) to the central bank.

• The government had issued a notification appointing August 9 as the date on which the relevant part

of that Finance Act, namely, Part VII of Chapter VI, shall come into effect. HFCs will henceforth be

treated as one of the categories of non-banking financial companies (NBFCs) for regulatory

purposes.

• Also, any housing finance institution, which is a company, desirous of making an application for

registration under sub-section 2 of section 29A of the NHB Act, 1987 (as amended by Act 23 of 2019)

must approach the department of Non- Banking Regulation at the RBI.

www.YouTube.com/SleepyClasses www.SleepyClasses.com

32

Jashanjot Sidhu
Jashanjot Sidhu
Jashanjot Sidhu

• The grievance redressal mechanism with regard to HFCs will also continue to be with the NHB.

52.Which of the following is/are correct in context of payments banks in India

1. They can issue only secured personal loans

2. Customers can open a savings account with deposit of only up to Rs 1 lac

3. They can’t accept fixed or recurring deposits

A. 1 and 2 only

B. 2 and 3 only

C. 1 only

D. All are correct

Answer: B

Explanation

• A payments bank aims to further financial inclusion, especially through savings accounts and

payments services.

• Accordingly, a payments bank is not allowed to give any form of loan or issue a credit card, which is

also a form of unsecured personal loan.

• Even in case of savings accounts, a payments bank has certain restrictions.

✓Customers can open a savings account with deposits of only up to ₹ 1 lakh, which is also the

maximum balance allowed.

✓These banks currently offer interest rates similar to that being offered by regular banks.

✓As per RBI guidelines, payments banks can‘t accept fixed or recurring deposits.

53.Which of the following is true in context of NIIF (National Investment and Infrastructure Fund)?

1. NIIF currently manages five funds each with its distinctive investment mandate.

2. The funds are registered as Alternative Investment Fund (AIF) with the Securities and Exchange

Board of India (SEBI).

Which of the following is/are correct in this context?

A. 1 only

B. 2 only

C. Both are correct

D. None is correct

Answer: B

www.YouTube.com/SleepyClasses www.SleepyClasses.com

33

Jashanjot Sidhu
Jashanjot Sidhu
Jashanjot Sidhu

Explanation

• NIIF currently manages three funds each with its distinctive investment mandate.

• The funds are registered as Alternative Investment Fund (AIF) with the Securities and Exchange

Board of India (SEBI).

• MASTER FUND: A fund focused on creating scalable sectoral platforms in core infrastructure.

• FUND OF FUNDS: A fund focused on anchoring and investing in credible and reputed third party

managers with a strong track record across diversified sectors within infrastructure services and

allied sectors.

• STRATEGIC FUND: A fund focused on investing in strategic assets and projects with longer term

horizon across various stages of development.

54.Which of the following is correct?

1. APEDA, under The Ministry of Agriculture, recommends increase in rate of customs duty on

imports of palm oil.

2. Any recommendation to increase duties becomes applicable after being notified by the Finance

Ministry.

A. 1 only

B. 2 only

C. Both are correct

D. None of the above

Answer: B

Explanation

• The Director General of Trade Remedies has recommended increase in rate of customs duty by 5

per cent on imports of two varieties of palmolein oil originating in Malaysia for a period of 180 days to

safeguard the interests of domestic industry.

• DGTR is under the Ministry of Commerce & Industry.

• The increased duties will be applicable once notified by the Finance Ministry.

• The alleged surge in imports happened following the lowering of import duties on the items from

Malaysia as part of the India-Malaysia Comprehensive Economic Cooperation Agreement

55.National List of Essential Medicines (NLEM) falls under the purview of

A. Ministry of Health & Family Welfare

B. Department of Pharmaceuticals under Ministry of Chemicals and Fertilizers

C. Ministry of Finance www.YouTube.com/SleepyClasses

www.SleepyClasses.com 34

Jashanjot Sidhu
Jashanjot Sidhu
Jashanjot Sidhu
Jashanjot Sidhu
Jashanjot Sidhu
Jashanjot Sidhu
Jashanjot Sidhu
Jashanjot Sidhu

D. NITI Aayog

Answer: A

Explanation

• As per the World Health Organisation (WHO), Essential Medicines are those that satisfy the priority

health care needs of the population.

• The list is made with consideration to disease prevalence, efficacy, safety and comparative cost-

effectiveness of the medicines.

• Ministry of Health and Family Welfare, Government of India hence prepared and released the first

National List of Essential Medicines of India in 1996 consisting of 279 medicines.

• The current NLEM, drawn in 2015, has 376 drugs listed under it.

• Hygiene products, such as adult diapers, sanitary napkins, soaps and floor disinfectants, are being

considered for inclusion as the list is being expanded.

• However, their inclusion in NLEM will not automatically lead to the product’s price control as there is

a proposal to delink the list from the purview of price control

56.Which of the following is correct in context of higher depreciation rates being used for cars

1. Resale value of vehicles will come down faster

2. It will bring down insurance premium in subsequent years

A. 1 only

B. 2 only

C. Both are correct

D. None of the above

Answer: C

Explanation

• The Finance Minister has announced doubling of depreciation rate to 30 per cent for vehicles bought

until March 31, 2020.

• A higher depreciation rate will mean that

✓The resale value will come down faster, which will prompt customers to replace the car sooner

✓Bringing down insurance premium from the second year

✓Allowing businesses to claim higher business expenses, which would lead to more tax benefits

www.YouTube.com/SleepyClasses www.SleepyClasses.com

35

Jashanjot Sidhu
Jashanjot Sidhu
Jashanjot Sidhu
Jashanjot Sidhu

57.Which of the following is correct in context of Good Agriculture Practices (GAP)

1. It defines appropriate methodology and inputs to be followed by farmers in order to produce safe

and high quality agri-products.

2. The Indian version of GAP is called INDGAP and is developed and managed by NITI AAYOG.

A. 1 only

B. 2 only

C. Both are correct

D. None of the above

Answer: A

Explanation

• GAP is a standard that defines appropriate methodology and inputs to be followed by farmers in

order to produce safe and high quality agri-products which is in demand throughout the world.

• The Indian version of GAP, called INDGAP, is developed and managed by Quality Council of India

(QCI).

• India is trying to harmonise standards of individual nations on Good Agriculture Practices (GAP)

among SAARC nations to boost Regional trade in south Asia.

58.Short-term and Long-term capital gains by FPIs and domestic investors are taxed. It is taxed as

A. A direct tax

B. An indirect tax

C. Both

D. None

Answer: A

Explanation

• Finance Minister announced the withdrawal of enhanced surcharge on short-term and long-term

capital gains by FPIs and domestic investors.

• Income arising from derivatives for the domestic investors including AIF category-III as well as for

foreign investors who are not FPIs, has been treated as business income and not as capital gains, and

taxed at applicable normal income tax rates.

• Income Tax Act, 1961 contains special provisions [section 115AD read with section 2(14) of the Act]

for taxation of income from derivatives in case of FPIs.

www.YouTube.com/SleepyClasses www.SleepyClasses.com

36

Jashanjot Sidhu
Jashanjot Sidhu
Jashanjot Sidhu

59.The International Coalition for Disaster Resilient Infrastructure will have its headquarters in

A. Bhuj, Gujarat

B. Machilipatnam, Andhra Pradesh

C. New Delhi, India

D. Bangkok, Thailand

Answer: C

Explanation

• The Union Cabinet has given ex-post facto approval for the Establishment of an International

Coalition for Disaster Resilient Infrastructure (CDRI) along with its supporting Secretariat Office in

New Delhi.

• The CDRI is proposed to be launched at the UN Climate Action Summit in New York, USA on 23rd

September 2019.

• The CDRI will serve as a platform where knowledge is generated and exchanged on different aspects

of disaster and climate resilience of infrastructure.

• Initiatives will work at the intersection of Sendai Framework, Sustainable Development Goals

(SDGs) and Climate Change Adaptation with a focus on infrastructure

• Publication of natural hazard risk information about the different regions in India will allow the

public to understand the risk in their regions and demand for risk mitigation and preparedness

measures from their local and State Government.

60.Which of the following is/are correct?

1. Under investor-state dispute settlement (ISDS) mechanism, domestic laws can only be challenged

in the domestic tribunals.

2. Rules of origin are used to implement measures and instruments of commercial policy such as

antidumping duties and safeguard measures.

A. 1 only

B. 2 only

C. Both are correct

D. None of the above

Answer: B

Explanation

• In RCEP, India‘s negotiating team is fighting against the proposed inclusion of the controversial

investor-state dispute settlement (ISDS) in the pact by members like Japan and Singapore.

• India is also arguing for a strict rules of origin (ROO) clause to stop third country imports.

www.YouTube.com/SleepyClasses www.SleepyClasses.com

37

Jashanjot Sidhu
Jashanjot Sidhu

• There are also demands of better offers in services and try and extend protection to a number of

sensitive dairy and farm items and industrial goods

• New Delhi also does not want an ISDS mechanism in RCEP as it does not want its domestic laws to be

challenged in offshore arbitral tribunals.

• Rules of origin are the criteria needed to determine the national source of a product. Their

importance is derived from the fact that duties and restrictions in several cases depend upon the

source of imports.

• Rules of origin are used:

✓to implement measures and instruments of commercial policy such as anti-dumping duties and

safeguard measures;

✓to determine whether imported products shall receive most favoured-nation (MFN) treatment or

preferential treatment;

✓for the purpose of trade statistics

✓for application of labelling and marking requirements

✓for government procurement

61.Motihari-Amlekhganj (India-Nepal) were recently in news because of inauguration of

A. A new cross-border mega multi-purpose dam

B. A new cross-border fuel pipeline

C. A new cross-border wildlife sanctuary

D. A new cross-border waterway route

Answer: B

Explanation

• The two countries inaugurated South Asia‘s first cross border fuel pipeline.

• It will help Kathmandu cut fuel prices by Rs 2 per litre on account of reduced transportation cost.

• The 69 km Motihari-Amlekhganj pipeline project was completed 15 months ahead of schedule.

• It can transport two million tonnes of petro-products annually to the landlocked Himalayan nation.

62.Which of the following is/are correct?

1. India‘s $1 billion Line of Credit to Russia is the first time India has extended it to any country

2. India‘s $1 billion Line of Credit to Russia will be primarily for the development of resource-rich far

west region of Russia

Which of the above statements are correct?

www.YouTube.com/SleepyClasses www.SleepyClasses.com

38

Jashanjot Sidhu
Jashanjot Sidhu
Jashanjot Sidhu
Jashanjot Sidhu

A. 1 only

B. 2 only

C. Both are correct

D. None of correct

Answer: D

Explanation

63.Which of the following is correct in context military expenditure

1. India is the second highest military spender in the world after USA.

2. USA has been the top military spender for over a 100 years.

Which of the following is/are correct in this context?

A. 1 only

B. 2 only

C. Both are correct

D. None of the above

Answer: D

www.YouTube.com/SleepyClasses www.SleepyClasses.com

39

Explanation

64.Which of the following is correct in context of investment in Sovereign Gold Bonds

1. Capital gains made by investing in these Bonds is tax exempt (provided they are held till maturity).

2. The minimum investment in these bonds is ten grams.

A. 1 only

B. 2 only

C. Both are correct

D. None of the above

Answer: A

www.YouTube.com/SleepyClasses www.SleepyClasses.com

40

Jashanjot Sidhu

Explanation

• SGBs are government securities denominated in grams of gold. They are substitutes for holding

physical gold.

• The Bond is issued by Reserve Bank on behalf of Government of India.

• The Bonds are issued in denominations of one gram of gold and in multiples thereof.

✓Minimum investment in the Bond shall be one gram with a maximum limit of subscription of 4 kg for

individuals.

• On maturity, the redemption price shall be based on simple average of closing price of gold of 999

purity of previous 3 business days from the date of repayment, published by the India Bullion and

Jewellers Association Limited.

• Capital gains made by investing in these Bonds is tax exempt, provided they are held till maturity.

65.The Unified Payments Interface (UPI) is managed by

A. RBI

B. National Payments Corporation of India (NPCI)

C. Ministry of Finance

D. Ministry of Commerce

Answer: B

Explanation

• National Payments Corporation of India (NPCI) is an umbrella organization for all retail payments in

India.

• It was set up with the guidance and support of the Reserve Bank of India (RBI) and Indian Banks

Association (IBA).

• The ten core promoter banks are State Bank of India, Punjab National Bank, Canara Bank, Bank of

Baroda, Union Bank of India, Bank of India, ICICI Bank, HDFC Bank, Citibank N. A. and HSBC.

• RuPay is also under the ambit of NPCI.

66.Which of the following is/are not true?

1. India‘s GDP per capita is amongst top 100 in the world.

2. India‘s GDP growth in dollar terms was faster as compared to GDP growth in rupee terms

A. 1 only

B. 2 only

C. Both 1 and 2

www.YouTube.com/SleepyClasses www.SleepyClasses.com

41

Jashanjot Sidhu
Jashanjot Sidhu
Jashanjot Sidhu

D. Neither 1 nor 2

Answer: C

Explanation

www.YouTube.com/SleepyClasses www.SleepyClasses.com

42

67.According to the UN‘s International Migrant Stock, which of the following country has the largest

population of diaspora

A. India

B. China

C. Mexico

D. Bangladesh

Answer: A

Explanation

• At 17.5 million, India‘s diaspora continues to be the largest in the world constituting 6.4% of the total

world migrant population of 272 million in mid-2019.

• After India, the largest number of migrants are from Mexico (11.8 million), China (10.7 million), and

Russia (10.5 million).

• While India‘s diaspora in absolute numbers has increased 10% from 15.9 million in 2015, as a share of

total world migrant population, it‘s remained largely static.

www.YouTube.com/SleepyClasses www.SleepyClasses.com

43

Jashanjot Sidhu
Jashanjot Sidhu
Jashanjot Sidhu

• The UAE, the US and Saudi Arabia—with 3.4 million, 2.6 million and 2.4 million respectively— were

the top three destinations for Indians.

68.Which of the following add(s) to the volatility of Rupee

1. Increased offshore trading in dollar-rupee forwards

2. Increased payments of imports in Rupees

A. 1 only

B. 2 only

C. Both 1 and 2

D. Neither 1 nor 2

Answer: A

Explanation

• According to the latest survey from the Bank for International Settlements (BIS):

✓London has overtaken Mumbai to become the top centre for trading the rupee.

✓Average daily volumes for rupee in the UK soared to $46.8 billion in April, released this week.

✓That exceeded the $34.5 billion recorded in India.

✓Points to the growing size of the offshore rupee market.

✓It could amplify currency volatility in the domestic currency

✓It also reduce the effectiveness of policy steps taken to limit volatility during times of stress

✓Trading in dollar-rupee offshore non-deliverable forwards increased threefold over the three-year

period.

• Rupee trading includes spot, outright forwards, foreign-exchange swaps, and other products.

• It also jumped in Singapore, Hong Kong and the US over the three-year period, according to BIS.

www.YouTube.com/SleepyClasses www.SleepyClasses.com

44

Jashanjot Sidhu
Jashanjot Sidhu
Jashanjot Sidhu
Jashanjot Sidhu
Jashanjot Sidhu
Jashanjot Sidhu
Jashanjot Sidhu

69.Country‘s first central police university will be set up by the Union Home Ministry in

A. Allahabad

B. Greater Noida

C. Mumbai

D. Hyderabad

Answer: B

Explanation

• The country‘s first central police university will be

set up by the Union home ministry off the Yamuna

Expressway in Greater Noida.

• Centre had given in-principle approval for the

varsity.

• The university will offer undergraduate and post-

graduate courses on policing, internal security,

cybercrime and forensic sciences.

• The country does not have a central university for

police yet, though states like Rajasthan, Gujarat

and Jharkhand have dedicated police varsities.

70.NITI Aayog has finalised a fresh list of 11 state run companies for strategic disinvestment. In this

context, Strategic Disinvestment refers to:

1. Transfer of management control

2. Transfer of ownership

A. 1 only

B. 2 only

C. Both

D. None

Answer: C

Explanation

• NITI Aayog has finalised a fresh list of 11 state run companies for strategic disinvestment. It will help

the Centre‘s reform drive to counter the economic slowdown and raise cash.

• NITI Aayog had earlier identified over three dozen companies for the strategic sale. Strategic sale

refers to transfer of management control and ownership.

www.YouTube.com/SleepyClasses www.SleepyClasses.com

45

Jashanjot Sidhu

• While representatives of NITI Aayog are included in the panel for the asset sale programme,

Department of Investment and Public Asset Management (DIPAM) will have a greater role in

prodding the administrative ministries to fast-track decision linked to state-run companies under

their watch.

71.Global Goalkeeper Award is given by

A. UNESCO

B. UNEP

C. UNDP

D. None of the above

Answer: D

Explanation

• Prime Minister Narendra Modi was honoured with the prestigious Global Goalkeeper Award by the

Bill and Melinda Gates Foundation for his leadership and commitment to the Swachch Bharat

Abhiyaan.

• The Foundation hosted the fourth annual Goalkeepers Global Goals Awards.

• The annual awards, in five categories, are presented to leaders and individuals for their efforts in

achieving the Sustainable Development Goals (SDGs).

• The categories are:

✓Progress

✓Change maker

✓Campaign

✓Goalkeepers Voice

✓The Global Goalkeeper

72.Often in news, gig economy, primarily refers to

A. Rise of Stand-up comedy as a career.

B. Rise of Politics as a career.

C. Rise of workers taking up short-term contracts or freelance work.

D. Rise of AI based careers

Answer: C

www.YouTube.com/SleepyClasses www.SleepyClasses.com

46

Jashanjot Sidhu
Jashanjot Sidhu
Jashanjot Sidhu
Jashanjot Sidhu

Explanation

• In a gig economy, temporary, flexible jobs are commonplace and companies tend toward hiring

independent contractors and freelancers instead of full-time employees.

• A gig economy undermines the traditional economy of fulltime workers who rarely change positions

and instead focus on a lifetime career.

• The government is to devise social security scheme for gig workers.

• The move forms a key part of the new Code on Social Security drafted by the government and is

aimed at ensuring the social security needs of gig workers.

• It could create a furore among gig economy giants, which have made freelance workers the backbone

of their business.

73.Which of the following have a Geographical Indication (GI) tag

1. Kandhamal turmeric

2. Panchamirtham of Vaishno Devi Temple

3. Dindigul lock

Which of the following is/are correct in this context?

A. 1 only

B. 1 and 3 only

C. 2 only

D. All of the above

Answer: B

Explanation

• GI tag is approved by the Geneva-headquartered World Trade Organization.

• A GI tag recognises the place of origin of a product and the specific qualities or means of production

associated with it.

• Acting as a certification, it is a way of ensuring that similar products from elsewhere cannot be sold

under this name.

• The tag is valid for a decade, after which it can be renewed for another 10 years.

• In India, the GI tag is governed by the Geographical Indication of Goods (Registration and Protection)

Act which came into being in 1999. That was the year Darjeeling tea became the country‘s first

product to bag a GI tag.

Panchamirtham of Palani Murugan Temple:

• Panchamirtham from Palani becoming the first prasad to clinch the honour last month.

www.YouTube.com/SleepyClasses www.SleepyClasses.com

47

Jashanjot Sidhu

• The panch in the name refers to the five key ingredients in the prasad — banana, jaggery, cow ghee,

cardamom and honey.

• Dates and sugar crystals are added for taste.

The Dindigul lock and Kandangi sari, also from TN, are traditional handmade products.

• Dindigul locks are world famous because of their durability, earning their city of its origin the moniker

lock city.

From elsewhere around the country

• 14 products made it to the list this year.

• The GI tag is granted by governments including Kandhamal haldi (turmeric) from Odisha, Himachali

kaala zeera (black cumin) from Himachal Pradesh, jeeraphool rice from Chhattisgarh, Sirsi supari

(betelnut) and Coorg Arabica coffee from Karnataka, Marayur jaggery from Kerala, and Himachali

Chulli (apricot) oil.

• Some of the well-known GI tags from around the world are Parmesan Reggiano cheese (Italy), Scotch

whisky (Scotland) and Champagne (France).

74.London Interbank Offered Rate (LIBOR) will soon be replaced by

A. PRIYA

B. NINJA

C. SONIA

D. ELISA

Answer: C

Explanation

London Interbank Offered Rate (LIBOR)

• The London Interbank Offered Rate (LIBOR) is a benchmark interest rate at which major global banks

lend to one another in the international interbank market for short-term loans.

• LIBOR is the average interest rate at which major global banks borrow from one another.

• It is based on five currencies including the US dollar, the euro, the British pound, the Japanese yen,

and the Swiss franc, and serves seven different maturities—overnight/spot next, one week, and one,

two, three, six, and 12 months.

• The combination of five currencies and seven maturities leads to a total of 35 different LIBOR rates

calculated and reported each business day. The most commonly quoted rate is the three months U.S.

dollar rate, usually referred to as the current LIBOR rate.

Sterling Overnight Index Average (SONIA)

• SONIA is based on actual transactions and reflects the average of the interest rates that banks pay to

borrow sterling overnight from other financial institutions. www.YouTube.com/SleepyClasses

www.SleepyClasses.com 48

Jashanjot Sidhu
Jashanjot Sidhu
Jashanjot Sidhu
Jashanjot Sidhu
Jashanjot Sidhu

• Financial businesses and institutions use SONIA in a variety of ways.

• The Bank of England runs SONIA – the risk-free rate for sterling markets.

75.Which of the following is correct in context of PM‘s Economic Advisory Council

1. It came into being in 2015 after Planning Commission was dismantled.

2. Though it is a non constitutional body, it is a Statutory body.

A. 1 only

B. 2 only

C. Both

D. None

Answer: D

Explanation

• The PMEAC came into existence over three and a half decades ago, against the backdrop of a difficult

economic situation. Prime Minister Indira Gandhi, who had returned to power in 1980, faced

formidable economic challenges.

www.YouTube.com/SleepyClasses www.SleepyClasses.com

49

Jashanjot Sidhu
Jashanjot Sidhu

• In the initial years of its existence, the members of the Council included the famed economist K N Raj,

besides C Rangarajan, who would later become the Governor of the Reserve Bank of India, and Vijay

Kelkar, who was the first Secretary of the PMEAC during 1982-83.

• Read More Here

76.In Economic parlance, a Promoter is

1. Who is in control of the organization

2. Who is instrumental in formulation of the company

A. 1 only

B. 2 only

C. Both

D. None of the above

Answer: B

Explanation

• SEBI (Issue of Capital and Disclosure Requirements) Regulations, define promoter and promoter

group as under:

✓the person or persons who are in control of the issuer;

✓the person or persons who are instrumental in the formulation of a plan or programme pursuant

to which specified securities are offered to public;

✓The person or persons named in the offer document as promoters.

• Provided that a director or officer of the issuer or a person, if acting as such merely in his

professional capacity, shall not be deemed as a promoter.

• Provided further that a financial institution, scheduled bank, foreign institutional investor and mutual

fund shall not be deemed to be a promoter merely by virtue of the fact that ten per cent or more of

the equity share capital of the issuer is held by such person.

• Provided further that such financial institution, scheduled bank and foreign institutional investor shall

be treated as promoter for the subsidiaries or companies promoted by them or for the mutual fund

sponsored by them.

www.YouTube.com/SleepyClasses www.SleepyClasses.com

50

Jashanjot Sidhu
Jashanjot Sidhu
Jashanjot Sidhu
Jashanjot Sidhu

77.Which of the following is/are true in context of Securities Appellate Tribunal

1. It is a statutory body

2. It also hears and disposes of appeals against orders passed by the Pension Fund Regulatory and

Development Authority (PFRDA)

A. 1 only

B. 2 only

C. Both

D. None

Answer: C

Explanation

• It is a statutory body established under the provisions of Section 15K of the Securities and Exchange

Board of India Act, 1992.

• It hears and disposes of appeals against orders passed by the:

✓Securities and Exchange Board of India

✓Pension Fund Regulatory and Development Authority (PFRDA)

✓Insurance Regulatory Development Authority of India (IRDAI)

78.Which of the following is/are true?

1. Core sector constitutes less than 40% weight in the Index of Industrial Production (IIP)

2. IIP data is compiled and published by CSO every month

A. 1 only

B. 2 only

C. Both

D. None of the above

Answer: B

Explanation

• Core sector constitutes more than 40% (in fact, 40.27%) of IIP

• Central Statistics Office (CSO) of the Ministry of Statistics and Programme Implementation (MoSPI)

compiles and releases the IIP data

✓It is classified in two ways: Sector-wise and Use based

• To learn more click here

www.YouTube.com/SleepyClasses www.SleepyClasses.com

51

Jashanjot Sidhu
Jashanjot Sidhu

79.In news in context of Mutual Funds, Standstill agreements, refer to

A. An understanding between a lender and a regulator wherein the lender stops demanding a

scheduled payment of interest or principal on a loan so that the borrower gets time to restructure its

liabilities.

B. An understanding between a lender and a borrower wherein the lender stops demanding a

scheduled payment of interest or principal on a loan so that the borrower gets time to restructure its

liabilities.

C. An understanding between a regulator and a borrower wherein the lender stops demanding a

scheduled payment of interest or principal on a loan so that the borrower gets time to restructure its

liabilities.

D. None of the above

Answer: B

Explanation

• Standstill Agreement is an understanding between a lender and a borrower, wherein the lender stops

demanding a scheduled payment of interest or principal on a loan so that the borrower gets time to

restructure its liabilities.

• While open-ended mutual fund schemes typically have a flexible redemption deadline, Closed end

schemes have strict fund closure deadlines linked to the maturity dates of the NCDs or other debt

papers, in which the scheme has invested in.

• When an AMC agrees to a standstill agreement, primarily to facilitate a borrower who is unable to

service debt on the due date, it essentially short-changes investors by disregarding the committed

FMP maturity date.

80.Which of the following is correct in context of National Company Law Tribunal?

1. Its Principal Bench is in Mumbai.

2. It is formed under the provisions of Companies Act, 2013.

A. 1 only

B. 2 only

C. Both

D. None

Answer: B

Explanation

• The Central Government constituted National Company Law Tribunal (NCLT) under section 408 of

the Companies Act, 2013 (18 of 2013) w.e.f. 01st June 2016.

• In the first phase the Ministry of Corporate Affairs set up eleven Benches.

www.YouTube.com/SleepyClasses www.SleepyClasses.com

52

Jashanjot Sidhu
Jashanjot Sidhu

• Principal Bench is at New Delhi.

• Ten other benches are at New Delhi, Ahmadabad, Allahabad, Bengaluru, Chandigarh, Chennai,

Guahati, Hyderabad, Kolkata and Mumbai.

81. Dearness Allowance of central government is calculated on the basis of All India

A. Consumer Price Index (Combined)

B. Consumer Price Index (Urban)

C. Consumer Price Index (Industrial Workers)

D. Consumer Price Index (Urban Non-Manual Employees)

Answer: C

Explanation

• The 7th Pay Commission had recommended not to change the formula for DA calculation.

• The DA is calculated on the basis of All India Consumer Price Index (Industrial Workers).

• The 7th Pay Commission noted, “The VI CPC had recommended that the National Statistical

Commission may be asked to explore the possibility of a specific survey covering government

employees exclusively, so as to construct a consumption basked representative of government

employees and formulate a separate index. This has, however, not been done”.

82.Which of the following Ministerial conferences of WTO is/are correctly matched with their host city:

1. 2015 Nairobi

2. 2017 Bali

3. 2020 Nur-Sultan, Kazakhstan

A. 1 only

B. 2 only

C. 1 & 3 only

D. All of the above

Answer: C

Explanation

The topmost decision-making body of the WTO is the Ministerial Conference, which usually meets every

two years.

Ministerial Conferences

1. Nur-Sultan, Kazakhstan, 8-11 June 2020

www.YouTube.com/SleepyClasses www.SleepyClasses.com

53

Jashanjot Sidhu

2. Buenos Aires, 10-13 December 2017

3. Nairobi, 15-19 December 2015

4. Bali, 3-6 December 2013

5. Geneva, 15-17 December 2011

6. Geneva, 30 November - 2 December 2009

7. Hong Kong, 13-18 December 2005

8. Cancún, 10-14 September 2003

9. Doha, 9-13 November 2001

10.Seattle, November 30 – December 3, 1999

11.Geneva, 18-20 May 1998

12.Singapore, 9-13 December 1996

83. Recently in news because of GI Tag, Tirur betel leaves, belong to

A. Tamil Nadu

B. Kerala

C. Telengana

D. Andhra Pradesh

Answer: B

Explanation

• Recently, Geographical Indication status has been received for Tirur Vettila, the unique betel leaf

variety from Malappuram district of Kerala.

• The scientists at the Kerala Agriculture University pointed out that Tirur betel leaves have

significantly high content of total chlorophyll and protein and more pungency.

• Eugenol is the major essential oil contributing to its pungency. The shelf life of this betel leaf is also

more compared with similar varieties.

84.Which of the following is correct in context of Global Competitiveness Report:

1. Each indicator, or “pillar” uses a scale from 0 to 100, to show how close an economy is to the ideal

state or “frontier” of competitiveness in that area.

2. The index maps the competitiveness landscape of 141 economies through 103 indicators

organized into 12 themes.

A. 1 only

www.YouTube.com/SleepyClasses www.SleepyClasses.com

54

B. 2 only

C. Both

D. None

Answer: C

Explanation

• The index maps the competitiveness landscape of 141 economies through 103 indicators organized

into 12 themes.

• Each indicator, using a scale from 0 to 100, shows how close an economy is to the ideal state or

“frontier” of competitiveness.

• The pillars, which cover broad socio-economic elements are:

✓Institutions

✓Infrastructure

✓ICT adoption

✓Macroeconomic stability

✓Health

✓Skills

✓Product market

✓Labour market

✓The financial system

✓Market size

✓Business dynamism and

✓Innovation capability

85.India had introduced the concept of ‘Significant Economic Presence’ (SEP) in its tax laws. It

corresponds to

A. Taxing oil companies

B. Taxing online companies

C. Taxing infrastructure developers

D. Taxing start-ups

Answer: B

Explanation

• Multinational enterprises (MNEs) such as Google, Facebook, Amazon and Netflix, which have a huge

consumer base in several countries but no physical presence.

• In a digital economy, companies can increasingly do business with customers in a country without

having a physical presence — this required creation of globally accepted international taxation norms.

• Currently, a ‘source’ country (from which revenue is derived) can tax an MNE only once a nexus is

established through a physical presence in the country (referred to as a permanent establishment, or

a PE). Thereafter, only the profits attributed to the PE can be taxed.

• Under new tax laws, ‘some’ profits and corresponding taxing rights will be re-allocated to countries

where the consumers (markets) are based.

www.YouTube.com/SleepyClasses www.SleepyClasses.com

55

Jashanjot Sidhu
Jashanjot Sidhu
Jashanjot Sidhu
Jashanjot Sidhu

• OECD recently proposed rules to determine where tax should be paid (nexus rules) and on what

portion of profits they should be taxed (profit-allocation rules).

• India had introduced the concept of ‘Significant Economic Presence’ (SEP) in tax laws via the 2018

Budget.

86.Which of the following is/are true in context of the latest Census of Livestock (20th):

1. Livestock population has decreased by 4.6 per cent since 2012.

2. Among the States, Punjab has the highest number of livestock.

A. 1 only

B. 2 only

C. Both

D. None

Answer: B

Explanation

• The provisional data of the 20th Livestock Census released

by the Department of Animal Husbandry and Dairying on

Wednesday showed that the livestock population

increased by 4.6 per cent, from 512.06 million in 2012.

• Besides, there was a spectacular 16.8 per cent increase in

the poultry population in the country to 851.81 million,

mainly on account of a 46 per cent rise in backyard poultry

birds, whose numbers have gone up to 317 million.

• Among the States, Uttar Pradesh has the highest number

of livestock of 67.8 million (68.7 million in 2012), followed

by Rajasthan 56.8 million (57.7 million), Madhya Pradesh:

40.6 million (36.3 million) and West Bengal: 37.4 million

(30.3 million).

www.YouTube.com/SleepyClasses www.SleepyClasses.com

56

Jashanjot Sidhu
Jashanjot Sidhu
Jashanjot Sidhu
Jashanjot Sidhu

87.Which of the following is/are correct in context of 2019 Hurun Global Unicorn List:

1. It is a ranking of the world’s billion-dollar tech ‘start-ups’ founded in the 2000s and not yet listed

on a public exchange.

2. India has the 3rd highest number of unicorns in the world.

A. 1 only

B. 2 only

C. Both

D. None

Answer: C

Explanation

• The Hurun Research Institute has released the inaugural Hurun Global Unicorn List 2019, a ranking

of the world’s billion-dollar tech ‘start-ups’ founded in the 2000s and not yet listed on a public

exchange.

• This is the first year of the list, and follows from the sixth quarterly Hurun China Unicorn Index.

• Valuations are a snapshot as of 30 June 2019.

• There are 494 unicorns in all, based in 25 countries and 118 cities. Set up seven years ago on average,

they are worth $3.4 billion on average and $1.7 trillion in total.

• China pipped the US to lead by 206 versus 203, and together they accounted for over 80 per cent of

the world’s unicorns. Europe has 35 unicorns.

• India ranked third

✓India was placed third with 21 unicorns, led by payments solutions platform One97

Communications ($10 billion) and followed by cab aggregator Ola Cabs ($6 billion); online educator

Byju’s ($6 billion) and travel-stay finder OYO Rooms ($5 billion).

• The UK was fourth with 13.

88.Which of the following is/are correct in context of Financial Action Task Force (on MoneyLaundering):

1. It was founded in 1989 on a G7 initiative.

2. FATF grey lists a country which it considers as a safe haven for terror funding and money

laundering.

3. So far, no country has been blacklisted.

A. 1 only

B. 1 and 2 only

C. 3 only

www.YouTube.com/SleepyClasses www.SleepyClasses.com

57

Jashanjot Sidhu
Jashanjot Sidhu
Jashanjot Sidhu
Jashanjot Sidhu
Jashanjot Sidhu

D. All are correct

Answer: B

Explanation

• The Financial Action Task Force is an intergovernmental organization founded in 1989 on the

initiative of the G7 to develop policies to combat money laundering.

• In 2001, its mandate was expanded to include terrorism financing

• So far, only two countries have been blacklisted, they are Iran and North Korea.

89.Which of the following might be counted as the sub-components of GOVERNMENT SPENDING (G)

component of GDP

1. Government expenditures on final goods

2. Government expenditures on final services

3. It includes salaries of public servants

4. It includes purchases of weapons for the military

5. It includes any transfer payments, such as social security or unemployment benefits

A. All except 2 and 3

B. All except 3 and 4

C. All except 4 Only

D. All except 5 Only

Answer: D

Explanation

• G (Government spending) is the sum of government expenditures on final goods and services. It

includes:

✓Salaries of public servants

✓Purchases of weapons for the military

✓Any investment expenditure by a government

• It does not include any transfer payments, such as social security or unemployment benefits.

90.In context of GDP Deflator, Consider the following:

1. It is the ratio between GDP at constant prices and GDP at current prices.

2. If GDP at current prices is equal to GDP at constant prices, GDP deflator will be 1.

www.YouTube.com/SleepyClasses www.SleepyClasses.com

58

Jashanjot Sidhu
Jashanjot Sidhu
Jashanjot Sidhu

3. It is believed to be a better measure of price behaviour because it covers all goods and services

produced in the country. 

A. 1 and 2 Only

B. 1 and 3 Only

C. 2 and 3 Only

D. All of the above

Answer: C

Explanation

GDP Deflator refers to the

• Ratio between GDP at current prices and GDP at constant prices. 

✓Thus, first statement is reversed.

• It is believed to be a better measure of price behaviour because it covers all goods and services

produced in the country.

• Also, because the weight of services has not been equitably accounted in the Indian ‘headline

inflation’, that is, inflation at WPI

91.Usually, in a Short Position in trading, the investor gains when

A. The prices of underlying stocks rise

B. The prices of underlying stocks fall

C. The prices of underlying stocks stay the same for a long period of time

D. The gains can’t be determined from the share price

Answer: B

Explanation

• A short position refers to a trading technique in which an investor sells a security with plans to buy it

later.

• Shorting is a strategy used when an investor anticipates the price of a security will fall in the short

term.

• In common practice, short sellers borrow shares of stock from an investment bank or other financial

institution, paying a fee to borrow the shares while the short position is in place.

92.Which of the following is/are correct?

1. Mahanagar Telephone Nigam Limited is a designated Navratna CPSE.

2. Bharat Sanchar Nigam Limited is a designated Maharatna CPSE. www.YouTube.com/SleepyClasses

www.SleepyClasses.com 59

Jashanjot Sidhu

A. 1 only

B. 2 only

C. Both

D. None

Answer: A

Explanation

• The Government of India categorizes Central Public Sector Enterprises (CPSEs) under three

categories of Navratna, Miniratna and Maharatna.

• This categorization is based on the factors such as turnover, net worth and net profit on the annual

basis and the presence of the company in the stock exchange as per the Securities and Exchange.

• BSNL is Miniratna company

93.Which of the following is/are correct in context of MSP (Minimum Support Price)?

1. It is announced by the Government of India at the beginning of the sowing season for certain

crops.

2. It is based on the basis of the recommendations of the Commission for Agricultural Costs and

Prices (CACP).

A. 1 only

B. 2 only

C. Both

D. None

Answer: C

Explanation

• MSP is a form of market intervention by the Government of India to insure agricultural producers

against any sharp fall in farm prices.

• The minimum support prices are announced by the Government of India at the beginning of the

sowing season for certain crops on the basis of the recommendations of the Commission for

Agricultural Costs and Prices (CACP).

www.YouTube.com/SleepyClasses www.SleepyClasses.com

60

• MSP is price fixed by Government of India to protect the producer - farmers - against excessive fall in

price during bumper production years.

• The major objectives are:

✓to support the farmers from distress sales

✓to procure food grains for public distribution.

• In case the market price for the commodity falls below the announced minimum price due to bumper

production and glut in the market, government agencies purchase the entire quantity offered by the

farmers at the announced minimum price.

94.Which of the following is/are correct about the import of Palm Oil in India

1. Malaysia has been the largest importer in the year so far in 2019-20.

2. Indonesia has been the largest importer in the year 2018-19.

A. 1 only

B. 2 only

C. Both are correct

D. None is correct

Answer: C

Explanation

www.YouTube.com/SleepyClasses www.SleepyClasses.com

61

95.Which of the following is/are true in context of RCEP (Regional Comprehensive Economic

Partnership)?

1. It will have a combined GDP more than that of BRICS countries put together.

2. It will have a combined population more than that of BRICS countries put together.

A. 1 Only

B. 2 Only

C. Both are correct

D. None is correct

Answer: C

Explanation

RCEP

16 Countries

GDP (PPP) - $49.5 trillion

Population – 3.4 billion

BRICS

5 Countries

GDP (PPP) - $40.55 trillion

Population - 3.1 billion people

96. Indian Basket (IB), also known as Indian Crude Basket, is weighted average of:

1. Dubai and Oman (sour) crude oil prices

2. Brent Crude (sweet) crude oil prices

3. West Texas Intermediate (WTI)

Select the correct option

www.YouTube.com/SleepyClasses www.SleepyClasses.com

62

A. 1 only

B. 1 and 2 only

C. 3 only

D. All of the above

Answer: B

Explanation

According to Petroleum Planning & Analysis Cell, the Indian basket of crude oil is a derived basket

comprising of:

• Sour grade (Oman & Dubai average)

• Sweet grade (Brent dated)

• These are processed in Indian refineries in the ratio of 74.77 to 25.23.

Brent

• Brent is low-density crude oil with low sulphur content (sweet).

• It is extracted from the North Sea and comprises of Brent Blend, Forties Blend, Oseberg and Ekofisk

crudes.

• Since it is low-density and low-sulphur oil, it is ideal for refining into diesel, petrol and other fuels.

Dubai/Oman

• Dubai/Oman crude oil is of slightly lower quality, putting it in sour grade.

• It is a basket of crude oil from Dubai, Oman or Abu Dhabi.

• The composition of Dubai/Oman and Brent in the Indian basket of crude oil is decided on the

processing capacity of oil companies.

• Dubai/Oman crude oil is popular in the Asian market.

WTI

• WTI is crude oil extracted from wells in the US.

• WTI is also light and sweet but since it is extracted in land-locked areas instead of sea, it becomes

difficult from the transportation point of view.

• The WTI crude oil is sent via pipeline to Cushing, Oklahoma.

97.Dedicated Freight Corridor Corporation of India Limited (DFCCIL) is a Public Sector Undertaking

(PSU) corporation under:

A. Ministry of Railways

B. Ministry of Road Transport and Highways

www.YouTube.com/SleepyClasses www.SleepyClasses.com

63

Jashanjot Sidhu

C. Ministry of Heavy Industries and Public Enterprises

D. None of these

Answer: A

Explanation

• Ministry of Railways initiated action to establish a Special Purpose Vehicle for construction, operation

and maintenance of the dedicated freight corridors.

• This led to the establishment of "Dedicated Freight Corridor Corporation of India Limited (DFCC)", to

undertake planning & development, mobilization of financial resources and construction,

maintenance and operation of the dedicated freight corridors.

• DFCC was incorporated as a company under the Companies Act 1956 on 30th October 2006

• The ministry of Railway has embarked upon a long term strategic plan to construct high capacity, high

speed, Dedicated Freight Corridors along the golden quadrilateral and its diagonals.

• In the first phase of this plan DFCCIL has been entrusted with the responsibility of construction,

maintenance and operation of two corridors – Eastern Corridor from Ludhiana to Dankuni with

Dadri-Khurja link and Western Corridor from Dadri to Jawaharlal Nehru Port - along with all

attached infrastructure, to enable Indian Railways & other qualified operators to run their freight

trains.

• DFCCIL’s role will primarily be that of the infrastructure provider with responsibility of construction,

operation and maintenance.

• The DFCCIL will accept freight trains on its system operate them on the DFC and then hand them

back to Indian Railways and other qualified operators at the other end.

98.Overarchingly, which of the following agreements of WTO ensure that your country’s consumers are

being supplied with food that is safe to eat?

A. Trade-Related Aspects of Intellectual Property Rights (TRIPS)

B. General Agreement on Trade in Services (GATS)

C. Agreement on the Application of Sanitary and Phytosanitary Measures (the "SPS Agreement“)

D. None is correct

Answer: C

Explanation

• The Agreement on the Application of Sanitary and Phytosanitary Measures (the "SPS Agreement")

entered into force with the establishment of the World Trade Organization on 1 January 1995.

• It concerns the application of food safety and animal and plant health regulations.

• It sets out the basic rules for food safety and animal and plant health standards.

• It allows countries to set their own standards. www.YouTube.com/SleepyClasses

www.SleepyClasses.com 64

Jashanjot Sidhu
Jashanjot Sidhu

• But it also says regulations must be based on science.

99. Which of the following is/are correct in context of Multidimensional Poverty Index?

1. It is released by UNDP.

2. HDI uses only single indicators for each dimension of poverty while MPI uses more than one

indicator for each one.

Select the correct option

A. 1 only

B. 2 only

C. Both

D. None

Answer: C

Explanation

• The MPI identifies overlapping deprivations that people experience across the same three

dimensions as the Human Development Index (health, education and standard of living) and shows

the proportion of people that are poor and the average number of deprivations each poor person

experiences at the same time.

• Using the United Nations Development Programme (UNDP) classification for developing regions, the

MPI 2019 covers 40 (out of 46) countries in Sub-Saharan Africa; 7 (out of 9) countries in South Asia;

11 (out of 24) in East Asia and the Pacific; 11 (out of 20) countries in the Arab States region; 20 (out of

33) countries in Latin America and the Caribbean; and 12 (out of 17) countries in Europe and Central

Asia.

• While both HDI and MPI use the 3 broad dimensions health, education and standard of living, HDI

uses only single indicators for each dimension of poverty while MPI uses more than one indicator

for each one.

• This, amongst other reasons, has led to the MPI only being calculated for just over 100 countries,

where data is available for all these diverse indicators, while HDI is calculated for almost all countries.

• It identifies how people are being left behind across three key dimensions:

✓health, education and living standards, and 10 indicators – Nutrition, child mortality, years of

schooling, school attendance, sanitation, cooking fuel, drinking water, electricity, housing and

assets. Those who are deprived in at least a third of the MPI’s components are defined as

multidimensionally poor. 

100.In context of Inclusive Growth, various 5-Year plans have been made and put into practice. Which of

the following 5-Year Plans is/are correctly matched with their stated titles:

1. Faster, More Inclusive and Sustainable Growth 12th Plan www.YouTube.com/SleepyClasses

www.SleepyClasses.com 65

Jashanjot Sidhu
Jashanjot Sidhu

2. Inclusive and Sustainable Growth 11th Plan

3. Inclusive Growth 10th Plan

Select the correct option

A. 1 Only

B. 1 and 2 Only

C. All are correct

D. None is correct

Answer: A

Explanation

10th Plan didn’t had any title.

1. Inclusive Growth 11th Plan (2007-2012)

2. Faster, More Inclusive and Sustainable Growth 12th Plan (2012-2017)

101. Which of the following is/are the functions of the Banks Board Bureau (BBB)

1. The Bureau will recommend for selection of heads - Public Sector Banks and Financial Institutions.

2. It will help Banks in developing strategies and capital raising plans.

A. 1 only

B. 2 only

C. Both are correct

D. None of the above

Answer: C

Explanation

• With a view to improve the Governance of Public Sector Banks (PSBs), the Government had decided

to set up an autonomous Banks Board Bureau.

• The Bureau will recommend for selection of heads - Public Sector Banks and Financial Institutions

and help Banks in developing strategies and capital raising plans.

• The Bureau which will have three ex-officio members and three expert members in addition to

Chairman.

• All the Members and Chairman will be part time.

• The BBB started functioning from 1st April, 2016.

www.YouTube.com/SleepyClasses www.SleepyClasses.com

66

Jashanjot Sidhu
Jashanjot Sidhu
Jashanjot Sidhu
Jashanjot Sidhu

102.Consider the following statements:

1. FERA (Foreign Exchange Regulation Act) was introduced in 1973.

2. FEMA (Foreign Exchange Management Act) came into existence in 1999.

3. FEMA also gave the RBI the authority to regulate capital account transactions.

Choose the correct option

A. 1 and 2 only

B. 3 only

C. 1 only

D. All of the above

Answer: A

Explanation

• When FERA was introduced in 1973, the Indian economy was suffering from an all-time low of

foreign exchange (forex) reserves. To rebuild these reserves, the government took a stance that all

forex earned by Indian residents — living within India or abroad — belonged to the Government of

India and had to be surrendered to the Reserve Bank of India (RBI).

✓FERA, thus, severely regulated all forex transactions that had a direct or indirect impact on India’s

forex reserves, which included the import and/or export of currency.

• RBI proposed FEMA in 1999 to administrate foreign trade and exchange transactions.

• FEMA would consolidate and amend the law relating to foreign exchange (forex) with the objective of

facilitating external trade and payments and for promoting the orderly development and

maintenance of foreign exchange market in India.

• FEMA also gave the RBI the authority to regulate capital account transactions.

103.In context of Consumer Food Price Index (CFPI), consider the following:

1. It is released by Central Statistics Office (CSO), Ministry of Statistics and Programme

Implementation (MOSPI).

2. It is released weekly.

Select the correct option

A. 1 only

B. 2 only

C. Both are correct

D. None is correct

Answer: A

www.YouTube.com/SleepyClasses www.SleepyClasses.com

67

Jashanjot Sidhu
Jashanjot Sidhu
Jashanjot Sidhu
Jashanjot Sidhu
Jashanjot Sidhu
Jashanjot Sidhu
Jashanjot Sidhu
Jashanjot Sidhu

Explanation

• Consumer Food Price Index (CFPI) is a measure of change in retail prices of food products consumed

by a defined population group in a given area with reference to a base year.

• The Central Statistics Office (CSO), Ministry of Statistics and Programme Implementation (MOSPI)

started releasing Consumer Food Price Indices (CFPI) for three categories:

✓Rural

✓Urban

✓Combined

• Separately on an all India basis with effect from May, 2014. Like Consumer Price Index (CPI), the CFPI

is also calculated on a monthly basis and methodology remains the same as CPI.

• The base year presently used is 2012.

• The CSO revised the Base Year of the CPI and CFPI from 2010=100 to 2012=100 with effect from

the release of indices for the month of January 2015.

104.Which of the following is/are the reforms that the States (State Agriculture Marketing Boards),

interested to integrate their APMCs/mandis with NAM (National Agricultural Market), are required

to carry out in their APMC Act

1. Single trading license (Unified) to be valid across the state.

2. Single point levy of market fee across the state.

3. Provision for e-auction/e-trading as a mode of price discovery.

Select the correct option

A. 2 only

B. 3 only

C. 2 and 3 only

D. All of the above

Answer: D

Explanation

• The eNAM electronic trading platform has been created with an investment by the Government of

India (through the Ministry of Agriculture & Farmers’ Welfare).

• It offers a “plug-in” to any market yard existing in a State (whether regulated or private).

• The special software developed for eNAM is available to each mandi which agrees to join the national

network free of cost with necessary customization to conform to the regulations of each State Mandi

Act.

www.YouTube.com/SleepyClasses www.SleepyClasses.com

68

Jashanjot Sidhu
Jashanjot Sidhu

• eNAM is not a parallel marketing structure but rather a device to create a national network of

physical mandis which can be accessed online.

• It seeks to leverage the physical infrastructure of the mandis through an online trading portal,

enabling buyers situated even outside the Mandi/State to participate in trading at theStates

interested to integrate their mandis with eNAM are required to carry out following three reforms in

their APMC Act:

A. Single trading license (Unified) to be valid across the state

B. Single point levy of market fee across the state; and

C. Provision for e-auction/e-trading as a mode of price discovery

• Ministry of Agriculture & Farmers' Welfare, Govt. of India has appointed Small Farmers'

Agribusiness Consortium (SFAC) as the Lead Implementing Agency of eNAM.

• SFAC will operate and maintain the eNAM platform with the help of a Strategic Partner, presently

NFCL.

105.In context of Index of Industrial Production, which of the following is/are correct?

1. Under ‘Electricity’ sector, electricity generation from renewable energy sources has been

excluded.

2. At sectoral level, Manufacturing has the highest number of item groups as well as weight.

Select the correct option

A. 1 Only

B. 2 Only

C. Both are correct

D. None is correct

Answer: B

Explanation

The following major changes have been effected in the coverage of 2011 series of IIP:

• The un-organised sector was included in the earlier series by including the value added of the same

for calculation of weights at NIC 2-digit level. Whereas the weights of the new series is calculated

based on the GVA of the organised manufacturing sector only.

• Mining sector basket now comprises of 29 minerals as opposed to 61 minerals in the old series. Under

‘Electricity’ sector, electricity generation from renewable energy sources has been included.

www.YouTube.com/SleepyClasses www.SleepyClasses.com

69

Jashanjot Sidhu
Jashanjot Sidhu
Jashanjot Sidhu
Jashanjot Sidhu
Jashanjot Sidhu
Jashanjot Sidhu
Jashanjot Sidhu
Jashanjot Sidhu
Jashanjot Sidhu

106.Which of the following is/are correct in context of BRICS

1. The first BRIC Summit was held in Beijing, China, in 2009.

2. South Africa joined the organization in 2015.

Select the correct option

A. 1 only

B. 2 only

C. Both are correct

D. None of the above

Answer: D

Explanation

• Originally the first four were grouped as "BRIC" (or "the BRICs"), before the induction of South Africa

in 2010.

• On November 30, 2001, Jim O’Neill, a British economist who was then chairman of Goldman Sachs

Asset Management, coined the term ‘BRIC’ to describe the four emerging economies of Brazil, Russia,

India, and China.The first BRIC Summit was held in Yekaterinburg, Russia, on June 16, 2009.

• In 2010, South Africa began efforts to join the BRIC grouping, and the process for its formal

admission began in August of that year.

• South Africa officially became a member nation on 24 December 2010, after being formally invited by

China to join and subsequently accepted by other BRIC countries.

• The group was renamed BRICS – with the "S" standing for South Africa – to reflect the group's

expanded membership.In April 2011, the President of South Africa, Jacob Zuma, attended the 2011

BRICS summit in Sanya, China, as a full member.

107.Which of the following is/are correct in context of G-20?

1. France as such is not a member because it is represented by EU.

2. United Kingdom is a separate member apart from EU.

A. 1 only

B. 2 only

C. Both are correct

D. None of the above is correct

Answer: B

www.YouTube.com/SleepyClasses www.SleepyClasses.com

70

Jashanjot Sidhu
Jashanjot Sidhu
Jashanjot Sidhu

Explanation

• The G20 (or Group of Twenty) is an international forum for the governments and central bank

governors from 19 countries and the European Union (EU).

• There are 20 members of the group: Argentina, Australia, Brazil, Canada, China, the European Union,

France, Germany, India, Indonesia, Italy, Japan, Mexico, Russia, Saudi Arabia, South Africa, South

Korea, Turkey, the United Kingdom, and the United States.

• Membership of the G20 consists of 19 individual countries plus the European Union. The EU is

represented by the European Commission and by the European Central Bank.

108.Often in news, Ways & Means Advances (WMA) is a temporary credit facility provided by the

A. World Bank

B. Reserve Bank of India

C. Ministry of Finance

D. IMF

Answer: B

Explanation

Ways and means advances (WMA) is a mechanism used by Reserve Bank of India (RBI) under its credit

policy to provide to States, banking with it, to help them tide over temporary mismatches in the cash

flow of their receipts and payments.

109.Numaligarh Refinery is located in:

A. Nagaland

B. Maharashtra

C. Assam

D. Karnataka

Answer: C

Explanation

• It is located in the Brahmaputra valley at Numaligarh in the district of Golaghat (Assam).

• It was established in accordance with the provisions made in the historic Assam Accord signed on

15th August 1985.

• NRL commenced commercial production in October, 2000.

• NRL has been conferred the status of Mini Ratna PSU.

www.YouTube.com/SleepyClasses www.SleepyClasses.com

71

Jashanjot Sidhu
Jashanjot Sidhu
Jashanjot Sidhu
Jashanjot Sidhu

110.Often used by the government for delivery of services to citizens, Common Service Centre (CSC), is a

special purpose vehicle (SPV) under the:

A. Ministry of Finance

B. Ministry of Electronics and Information Technology 

C. Ministry of Rural Development

D. Ministry of Women and Child Development

Answer: B

Explanation

• CSCs are the access points for delivery of:

✓Essential public utility services

✓Social welfare schemes

✓Healthcare services

✓Financial services

✓Education services

✓Agriculture services

• Apart from host of B2C services to citizens in rural and remote areas of the country.

www.YouTube.com/SleepyClasses www.SleepyClasses.com

72

• CSC e-Governance Services India Limited, a Special Purpose Vehicle, has been set up by the Ministry

of Electronics & IT under the Companies Act, 1956 to oversee implementation of the CSC scheme.

• CSC SPV provides a centralized collaborative framework for delivery of services to citizens through

CSCs, besides ensuring systemic viability and sustainability of the Scheme.

111.Which of the above is/are correct?

1. A bullet bond is a debt instrument whose entire principal value is paid all at once on the maturity

date.

2. Bullet bonds cannot be redeemed early by an issuer.

A. 1 only

B. 2 only

C. Both are Correct

D. None is Correct

Answer: C

Explanation

A bullet bond is:

1. A debt instrument whose entire principal value is paid all at once on the maturity date, as opposed

to amortizing the bond over its lifetime. 

2. Bullet bonds cannot be redeemed early by an issuer, which means they are non-callable.

112.In context of SAARC Currency Swap Facility, consider the following statements:

1. SAARC Currency Swap Facility came into operation in 1975.

2. The present swap arrangement is of overall corpus of US $2 billion.

3. The drawals can be made in any of the currencies of the SAARC Country.

Which of the above is/are incorrect?

A. 1 and 2 only

B. 2 and 3 only

C. 1 and 3 only

D. All of the above

Answer: C

www.YouTube.com/SleepyClasses www.SleepyClasses.com

73

Jashanjot Sidhu
Jashanjot Sidhu
Jashanjot Sidhu
Jashanjot Sidhu
Jashanjot Sidhu

Explanation

SAARC Currency Swap Facility 

• To further financial stability and economic cooperation within the SAARC region, the Reserve Bank of

India, with the concurrence of the Government of India, has decided to put in place a revised

Framework on Currency Swap Arrangement for SAARC countries 2019-2022.

• The Framework is valid from November 14, 2019 to November 13, 2022.

• Based on the terms and conditions of the Framework, the RBI would enter into bilateral swap

agreements with SAARC central banks, who want to avail swap facility.

• It may be recalled that the SAARC Currency Swap Facility came into operation on November 15,

2012 with an intention to provide a backstop line of funding for short term foreign exchange

liquidity requirements or balance of payment crises till longer term arrangements are made.

• Under the Framework for 2019-22, RBI will continue to offer swap arrangement within the overall

corpus of US $2 billion.

• The drawals can be made in US Dollar, Euro or Indian Rupee.

• The Framework provides certain concessions for swap drawals in Indian Rupee.

• The Currency Swap Facility will be available to all SAARC member countries, subject to their signing

the bilateral swap agreements.

113.Data and Statistics about employment are important for any country’s policy making. In this regard,

consider the following statement:

1. National Statistical Office (NSO) is a wing in the Ministry of Statistics and Programme

Implementation.

2. Labour force refers to the part of the population which supplies or offers to supply labour for

pursuing economic activities for the production of goods and services and, therefore, includes

both ‘employed’ and ‘unemployed’ persons.

Which of the above is/are correct?

A. 1 only

B. 2 only

C. Both are correct

D. None is correct

Answer: C

Explanation

• NSO is a wing of the Ministry of Statistics and Programme Implementation.

www.YouTube.com/SleepyClasses www.SleepyClasses.com

74

Jashanjot Sidhu
Jashanjot Sidhu
Jashanjot Sidhu
Jashanjot Sidhu
Jashanjot Sidhu
Jashanjot Sidhu

• Labour force refers to the part of the population which supplies or offers to supply labour for

pursuing economic activities for the production of goods and services and, therefore, includes both

‘employed’ and ‘unemployed’ persons.

• The LFPR is defined as the percentage of population in the labour-force.

• Labour-force according to current weekly status (CWS) is the number of persons either employed or

unemployed on an average in a week of 7 days preceding the date of survey.

• Recent survey showed that of the total workforce 37.7 per cent were self-employed, while 50 per

cent were salaried or regular workers and 12.4 per cent were casual labourers.

114.As per the WTO, in 2018:

1. India’s share in global exports for merchandise was less than 1 percent.

2. India’s share in global imports for merchandise was more than 2 per cent.

3. For the service sector, India’s share in global exports was more than 3 per cent

4. For the service sector, India’s share in global imports was less than 3 per cent.

Which of the above is/are correct?

A. All except 1

B. 2 and 3 only

C. All are correct

D. Only 2 is correct

Answer: B

Explanation

As per the WTO data released in April for 2018:

• India’s share in global exports for merchandise was 1.7 per cent and in global imports was 2.6 per

cent.

• For the service sector, its share in global exports was 3.5 per cent and an import was 3.2 per cent in

the last year.

115.Gig economy refers to:

A. Small numbers of people work full-time or in a permanent position.

B. Small numbers of people work part-time or in a temporary position.

C. Large numbers of people work full-time or in a permanent position.

D. Large numbers of people work part-time or in a temporary position

Answer: D www.YouTube.com/SleepyClasses

www.SleepyClasses.com 75

Jashanjot Sidhu
Jashanjot Sidhu
Jashanjot Sidhu
Jashanjot Sidhu
Jashanjot Sidhu
Jashanjot Sidhu
Jashanjot Sidhu
Jashanjot Sidhu
Jashanjot Sidhu
Jashanjot Sidhu
Jashanjot Sidhu

Explanation

• Large numbers of people work part-time or in a temporary position. It is a mega trend these days.

There is a sociological push across the globe towards flexibility in:

✓Working hours

✓Working spaces

✓Duration of employment.

• The emergence of marketplaces where in any part of the world, anyone can post his/her availability

for a project and put his/her capabilities up for grabs will create a pressure on the traditional way of

looking at flexi staffing.

116.Which one of the following is not a sub-index of the World Bank’s ‘Ease of Doing Business Index’?

A. Maintenance of law and order

B. Paying taxes

C. Registering property

D. Dealing with construction permits

Answer: A

117.With reference to land reforms in independent India, which one of the following statements is

correct?

A. The ceiling laws were aimed at family holdings and not individual holdings.

B. The major aim of land reforms was providing agricultural land to all the landless.

C. It resulted in cultivation of cash crops as a predominant form of cultivation.

D. Land reforms permitted no exemptions to the ceiling limits.

Answer: B

118.Which of the following adopted a law on data protection and privacy for its citizens known as

‘General Data Protection Regulation’ in April 2016 and started implementation of it from 25th May,

2018?

A. Australia

B. Canada

C. The European Union

D. The United States of America

Answer: C www.YouTube.com/SleepyClasses

www.SleepyClasses.com 76

Jashanjot Sidhu
Jashanjot Sidhu

119.With reference to India’s Five-Year Plans, which of the following statements is/are correct?

1. From the Second Five-Year Plan, there was a determined thrust towards substitution of basic and

capital good industries.

2. The Fourth Five-Yer Plan adopted the objective of correcting the earlier trend of increased

concentration of wealth and economic power.

3. In the Fifth Five-Year Plan, for the first time, the financial sector was included as an integral part of

the Plan.

Select the correct answer using the code given below

A. 1 and 2 only

B. 2 only

C. 3 only

D. 1, 2 and 3

Answer: D

120.The Global Competitiveness Report is published by the

A. International Monetary Fund

B. United Nations Conference on Trade and Development

C. World Economic Forum

D. World Bank

Answer: C

121.Which of the following is/are not one of the key dimensions used in WEF’s Gender Gap Report?

A. Economic Participation and Opportunity

B. Educational Attainment

C. Reproductive Health

D. Political Empowerment

Answer: C

Explanation

Adding some more information here:

• WEF’s GGR has 4 Dimensions.

• They further have sub-dimensions.

www.YouTube.com/SleepyClasses www.SleepyClasses.com

77

122.Which of the following indices are correctly matched?

1. Gender Inequality Index (GII) UNDP

2. Gender Related Development Index IMF

3. Social Institutions and Gender Index (SIGI) OECD Development Centre

4. Gender Parity Index (GPI) UNESCO

A. 1 & 4 only

B. 1 only

C. 1, 3 & 4 only

D. All of the above

Answer: C

Explanation

• Gender Inequality Index (GII) is an index for measurement of gender disparity that was introduced in

the 2010 Human Development Report 20th anniversary edition by the United Nations Development

Programme (UNDP).

• Gender Related Development Index (GDI) is an index designed to measure gender equality.

• GDI together with the Gender Empowerment Measure (GEM) were introduced in 1995 in the Human

Development Report written by the United Nations Development Program. The aim of these

measurements was to add a gender-sensitive dimension to the Human Development Index (HDI).

• In 2007 the OECD Development Centre introduced a Social Institutions and Gender Index (SIGI), a

composite indicator of gender equality that solely focuses on social institutions that affect the

equality between men and women, as well as on the four dimensions of family code, physical integrity,

ownership rights and civil liberties.

• Social institutions comprise norms, values and attitudes that exist in a society in relation to women.

• Gender Parity Index (GPI) is a socioeconomic index usually designed to measure the relative access to

education of males and females. This index is released by UNESCO. www.YouTube.com/SleepyClasses

www.SleepyClasses.com 78

Jashanjot Sidhu
Jashanjot Sidhu
Jashanjot Sidhu
Jashanjot Sidhu
Jashanjot Sidhu
Jashanjot Sidhu
Jashanjot Sidhu
Jashanjot Sidhu

• In its simplest form, it is calculated as the quotient of the number of females by the number of males

enrolled in a given stage of education (primary, secondary, etc.).

• A GPI equal to one signifies equality between males and females.

123.In context of Goods and Services Tax Compensation Fund, which of the following is/are correct?

1. It is a non-lapsable fund.

2. It forms part of public account of India.

Select the Correct Option

A. 1 only

B. 2 only

C. Both are correct

D. None is correct

Answer: C

Explanation

What is Compensation Cess

• It was introduced as relief for States for the loss of revenues arising from the implementation of GST.

• Since States’ local levies were subsumed under the GST, they were guaranteed a 14 per cent tax

revenue growth in the first five years after GST implementation by the Central government.

• States’ tax revenue as of FY16 is considered as the base year for the calculation of this 14 per cent

growth.

• Any shortfall against it is supposed to be compensated by the Centre using the funds specifically

collected as compensation cess.

From Where does Compensation Cess get Money

• Compensation cess is levied on five products considered to be ‘sin’ or luxury goods.

• For example, SUV vehicles (more than 4 metres) are charged 50 per cent GST, of which the GST tax

rate is 28 per cent and the compensation cess is 22 per cent.

• The collected compensation cess flows into the Consolidated Fund of India, and then transferred to

the Public Account of India, where a GST compensation cess account has been created.

• States are compensated bi-monthly from the accumulated funds in this account.

Other important things about Compensation cess

• The proceeds of the Compensation cess shall be credited to a non-lapsable Fund known as the Goods

and Services Tax Compensation Fund.

• It forms part of the public account of India.

www.YouTube.com/SleepyClasses www.SleepyClasses.com

79

Jashanjot Sidhu
Jashanjot Sidhu
Jashanjot Sidhu
Jashanjot Sidhu
Jashanjot Sidhu
Jashanjot Sidhu

• The input tax credit in respect of compensation cess on supply of goods or services can be utilised

only towards payment of the compensation cess on supply of goods or services.

• This cess will not be payable by exporters and those persons who have opted for compensation levy

124.National Company Law Appellate Tribunal (NCLAT) can hear appeals against the orders passed by:

1. National Company Law Tribunal(s) (NCLT)

2. Insolvency and Bankruptcy Board of India

3. Competition Commission of India (CCI)

Select the Correct Option

A. 1 only

B. 1 & 2 only

C. 1 & 3 only

D. All of the above

Answer: D

Explanation

• National Company Law Appellate Tribunal (NCLAT) was constituted under Section 410 of the

Companies Act, 2013 for hearing appeals against the orders of National Company Law Tribunal(s)

(NCLT), with effect from 1st June, 2016.

• NCLAT is also the Appellate Tribunal for hearing appeals against the orders passed by NCLT(s) under

Section 61 of the Insolvency and Bankruptcy Code, 2016 (IBC), with effect from 1st December, 2016.

• NCLAT is also the Appellate Tribunal for hearing appeals against the orders passed by Insolvency and

Bankruptcy Board of India under Section 202 and Section 211 of IBC.

• NCLAT is also the Appellate Tribunal to hear and dispose of appeals against any direction issued or

decision made or order passed by the Competition Commission of India (CCI) – as per the amendment

brought to Section 410 of the Companies Act, 2013 by Section 172 of the Finance Act, 2017, with

effect from 26th May, 2017.

125.The economic cost of food grains to the Food Corporation of India is Minimum Support Price and

bonus (if any) paid to the farmers plus (UPSC Prelims 2019)

A. transportation cost only

B. interest cost only

C. procurement incidentals and distribution cost

D. procurement incidentals and charges of godowns

Answer: C www.YouTube.com/SleepyClasses

www.SleepyClasses.com 80

Jashanjot Sidhu
Jashanjot Sidhu
Jashanjot Sidhu
Jashanjot Sidhu
Jashanjot Sidhu
Jashanjot Sidhu
Jashanjot Sidhu
Jashanjot Sidhu
Jashanjot Sidhu
Jashanjot Sidhu

Explanation

126.Recently in news, Operation Twist is related with:

A. Better risk assessment of securities

B. Enhancing the security of digital transactions

C. Changing the shape of the yield curve

D. Program to lower the money laundering activities

Answer: C

Explanation

• Operation Twist is the name given to a monetary policy tool that the US Federal Reserve had initiated

to influence the prevailing rate of interest in the markets.

• The tool essentially aims at changing the shape of the yield curve (hence the name — twist) through

simultaneous buying and selling of long- and short-term government bonds.

• In India, the RBI put through its version of Operation Twist by buying ₹10,000 crore worth of 10-year

government bonds while selling four shorter-term government bonds adding up to the same value.

• The intent is to moderate high long-term interest rates in the market and bring them closer to the

repo rate.

• High market yields on the 10-year G-sec influence bank lending rates on vehicle, housing and other

long-term loans, hurting retail borrowers.

• Post the announcement, the 10-year G-sec yield dropped by 20 basis points on intra-day trade to 6.6

per cent while the yields on shorter tenure bond (five year) rose 16 basis points to 6.67 per cent,

making for a flatter yield curve.

• That also helps bring down borrowing costs for the government.

www.YouTube.com/SleepyClasses www.SleepyClasses.com

81

127.Which of the following is seen as a sign of a coming recession?

A. Positively sloped Yield Curve

B. An inverted yield curve

C. A flat yield curve

D. A humped yield curve

Answer: B

Explanation

• A yield curve is the graph you get by plotting the interest rates at which a single borrower can take

loans from the market, for different time periods.

• Usually, to plot a yield curve, a graph is drawn through current market interest rates on 91-day, 182-

day and 364-day treasury bills, and then on 10-year, 20-year and 30-year borrowings.

• Most market watchers use the spread between one-year and 10-year borrowings to gauge the shape

of a country’s yield curve.

Example

• Indian Case: Positively Sloped Yield Curve

• The current yield on the Indian government’s one-year borrowings is 5.1 per cent, while that on its

10-year borrowing is at 6.6 per cent, making for an upward sloping yield curve.

• USA’s Case: Flat Yield Curve

• The government is borrowing one-year money at 1.6 per cent and 10-year money at 1.8 percent.

• The yield curve can tell a lot about the future health of the economy.

• A steeply upward sloping yield curve, like the one we’re seeing in India, is a sign that markets expect

interest rates to spike up sharply in future.

www.YouTube.com/SleepyClasses www.SleepyClasses.com

82

• The increase in future rates can come about due to several factors — inflation shooting up, the

economy reviving and upping the demand for money, or the markets perceiving a higher risk

associated with government borrowings due to a fiscal deficit overshoot.

• An upward sloping yield curve like India’s is however seen as quite desirable, compared to the

‘inverted yield curve’.

• An inverted yield curve is seen as a sign of an economy that is heading into a tailspin, because it is

usually a shrinking economy that prompts aggressive rate cuts.

• logic demands that lenders demand higher rates for giving out 10-year loans as opposed to one-year

loans.

• When they’re willing to lend 10-year money at lower rates, it’s a sign that they expect deflation.

128.Indian Highways Management Company Limited (IHMCL), is implementing FASTag across the nation

with the help of various issuer agencies as well as select banks. IHMCL is a company incorporated by:

1. National Highways Authority of India (NHAI)

2. National Payment Corporation of India (NPCI)

3. Unique Identification Authority of India (UIDAI)

Select the Correct Option

A. 1 only

B. 2 only

C. Both 1 & 2

D. All of the above

Answer: C

Explanation

• Indian Highways Management Company Limited (IHMCL) is a company incorporated by National

Highways Authority of India) and National Payment Corporation of India (NPCI).

• They are implementing this program with help from Toll Plaza Concessionaires, FASTag Issuer

Agencies and Toll Transaction Acquirer (select banks).

129.Which of the following is correct in context of Atal Bhujal Yojana (ATAL JAL):

1. It is a pan-India scheme running across all the States and UTs

2. Asian Development Bank is a major partner (bearing 50% of the cost)

Select the Correct Option

A. 1 only

www.YouTube.com/SleepyClasses www.SleepyClasses.com

83

B. 2 only

C. Both

D. None

Answer: D

Explanation

• ATAL JAL has been designed with the principal objective of strengthening the institutional framework

for participatory groundwater management and bringing about behavioral changes at the community

level for sustainable groundwater resource management in seven States, viz. Gujarat, Haryana,

Karnataka, Madhya Pradesh, Maharashtra, Rajasthan and Uttar Pradesh

• Implementation of the scheme is expected to benefit nearly 8350 Gram Panchayats in 78 districts in

these States.

• ATAL JAL will promote panchayat led ground water management and behavioural change with

primary focus on demand side management.

• Out of the total outlay of Rs. 6000 crore to be implemented over a period of 5 years (2020-21 to

2024-25), 50% shall be in the form of World Bank loan, and be repaid by the Central Government.

• The remaining 50% shall be through Central Assistance from regular budgetary support.

• The entire World Bank's loan component and Central Assistance shall be passed on to the States as

Grants.

130.Financial Stability Report, a very important report released in India, is released by an important

Indian institution. That institution is:

A. RBI

B. SEBI

C. Ministry of Finance

D. None of the above

Answer: A

Explanation

• The Reserve Bank of India releases the Financial Stability Report (FSR).

• The FSR reflects the collective assessment of the Sub-Committee of the Financial Stability and

Development Council (FSDC) on risks to financial stability, as also the resilience of the financial

system.

• The Report also discusses issues relating to development and regulation of the financial sector

www.YouTube.com/SleepyClasses www.SleepyClasses.com

84

Jashanjot Sidhu
Jashanjot Sidhu
Jashanjot Sidhu
Jashanjot Sidhu
Jashanjot Sidhu
Jashanjot Sidhu
Jashanjot Sidhu
Jashanjot Sidhu

131.Which of the following is/are correct?

1. India is the largest importer of gold in the World.

2. Dip in gold imports helps in narrowing the country’s trade deficit.

Select the Correct Option

A. 1 Only

B. 2 Only

C. Both are Correct

D. None is Correct

Answer: C

Explanation

• India is the largest importer of gold, which mainly caters to the demand of the jewellery industry.

• India’s gold imports, which have a bearing on the current account deficit (CAD).

• In volume terms, the country imports 800-900 tonnes of gold annually.

• To mitigate the negative impact of gold imports on trade deficit and CAD, the government increased

the import duty on gold to 12.5 per cent from 10 per cent in this year’s Budget.

• According to industry experts, businesses in the sector are shifting their manufacturing bases to

neighbouring countries due to high duty.

132.Merchant Discount Rate refers to:

A. The total discount a merchant offers on online transactions.

B. The sum total of all the charges and taxes that a digital payment entails.

C. The total discount a bank offers to the merchants for promoting online transactions.

D. None of the above

Answer: B

Explanation

Merchant Discount Rate

• Alternatively referred to as the Transaction Discount Rate or TDR is the sum total of all the charges

and taxes that a digital payment entails

• The MDR includes bank charges, which a bank charges customers and merchants for allowing

payments to be made digitally.

• Similarly, MDR also includes the processing charges that a payments aggregator has to pay to online

or mobile wallets or indeed to banks for their service.

www.YouTube.com/SleepyClasses www.SleepyClasses.com

85

Jashanjot Sidhu
Jashanjot Sidhu

• MDR is the cost paid by a merchant to a bank for accepting payment from their customers via digital

means, which is usually recovered from the customer. The merchant discount rate is expressed in

percentage of the transaction amount.

• In the budget, the government has mandated that neither the customers nor the merchants will have

to pay the so-called Merchant Discount Rate (or MDR) while transacting digital payments.

133.Which of the following is/are correct in context to SDG Index?

1. The Index has been constructed spanning across 15 out of 17 SDGs with a qualitative assessment

on SDG 16 and 17.

2. It has a set of 100 National Indicators derived from the National Indicator Framework.

Select the Correct Option

A. 1 only

B. 2 only

C. Both 1 & 2

D. None of the above

Answer: B

Explanation

• The SDG India Index is developed by NITI Aayog.

• The Index has been constructed spanning across 16 out of 17 SDGs with a qualitative assessment on

SDG 17.

• It tracks the progress of all the States and Union Territories (UTs) on a set of 100 National Indicators

derived from the National Indicator Framework, measuring their progress on the outcomes of

interventions and schemes of the Government of India.

• It has been designed to provide an aggregate assessment of the performance of all Indian States and

UTs, and to help leaders and change makers evaluate their performance on social, economic and

environmental parameters.

• The Index will be useful to States/UTs in assessing their progress on the SDGs in the following ways:

✓Support States/UTs to assess their progress against national targets and performance of their

peers to understand reasons for differential performance and devise better strategies to achieve

the SDGs by 2030. 

✓Support States/UTs to identify priority areas in which they need to invest and improve by enabling

them to measure incremental progress. 

✓Highlight data gaps related across SDGs for India to develop its statistical systems at the national

and State levels.

www.YouTube.com/SleepyClasses www.SleepyClasses.com

86

Jashanjot Sidhu
Jashanjot Sidhu
Jashanjot Sidhu
Jashanjot Sidhu

134.Which of the following is correct in context of BSE SENSEX?

1. It is a free-float market-weighted stock market index.

2. It comprises of 30 well-established and financially sound companies listed on Bombay Stock

Exchange.

Select the Correct Option

A. 1 only

B. 2 only

C. Both

D. None

Answer: C

Explanation

• The BSE SENSEX (also known as the S&P Bombay Stock Exchange Sensitive Index or simply the

SENSEX) is a free-float market-weighted stock market index of 30 well-established and financially

sound companies listed on Bombay Stock Exchange.

• The 30 component companies which are some of the largest and most actively traded stocks are

representative of various industrial sectors of the Indian economy.

• SENSEX is a stock market index whose components are weighted according to the total market value

of their outstanding shares.

• Every day an individual stock's price changes and thereby changes a stock index's value.

• The base value of the SENSEX was taken as 100 on 1 April 1979 and its base year as 1978–79.

• On 25 July 2001 BSE launched DOLLEX-30, a dollar-linked version of the SENSEX.

135.If the inflation in an economy is rising steadily, the central bank might:

A. Increase the repo rate

B. Decrease the repo rate

C. Keep the repo rate unchanged

D. None of the above

Answer: A

Explanation

• In the event of inflation, central banks increase repo rate as this acts as a disincentive for banks to

borrow from the central bank.

• This ultimately reduces the money supply in the economy and thus helps in arresting inflation.

www.YouTube.com/SleepyClasses www.SleepyClasses.com

87

136.Which of the following is/are correct?

1. India's coal sector was nationalised in 1951.

2. India is a net coal exporter.

Select the Correct Option

A. 1 Only

B. 2 Only

C. Both are Correct

D. None is Correct

Answer: D

Explanation

• Indian Coal &Imports

• India has the world's fourth-largest coal reserves.

• India imported 235 million tonnes of coal (for Rs1.71 lakh crore) last fiscal.

• Of these imports, about 100 million tonnes is non-substitutable as they are tied to the power plant or

user factories.

• The rest can be cut down.

• India’s coal sector was nationalised in 1973.

137.Which of the following act was amended to provide a statutory basis for the implementation of the

flexible inflation targeting (FIT) framework?

A. Banking Regulation Act, 1949.

B. Reserve Bank of India (RBI) Act, 1934

C. Deposit Insurance and Credit Guarantee Corporation Act, 1961.

D. None of the above.

Answer: B

Explanation

• The primary objective of monetary policy is to maintain price stability while keeping in mind the

objective of growth.

• Price stability is a necessary precondition to sustainable growth.

• In May 2016, the Reserve Bank of India (RBI) Act, 1934 was amended to provide a statutory basis for

the implementation of the flexible inflation targeting framework.

www.YouTube.com/SleepyClasses www.SleepyClasses.com

88

Jashanjot Sidhu
Jashanjot Sidhu
Jashanjot Sidhu
Jashanjot Sidhu
Jashanjot Sidhu
Jashanjot Sidhu

• The amended RBI Act also provides for the inflation target to be set by the Government of India, in

consultation with the Reserve Bank, once in every five years.

• Accordingly, the Central Government has notified in the Official Gazette 4 per cent Consumer Price

Index (CPI) inflation as the target for the period from August 5, 2016 to March 31, 2021 with the

upper tolerance limit of 6 per cent and the lower tolerance limit of 2 per cent.

• The Central Government notified the following as factors that constitute failure to achieve the

inflation target:

✓the average inflation is more than the upper tolerance level of the inflation target for any three

consecutive quarters; or

✓The average inflation is less than the lower tolerance level for any three consecutive quarters.

• Prior to the amendment in the RBI Act in May 2016, the flexible inflation targeting framework was

governed by an Agreement on Monetary Policy Framework between the Government and the

Reserve Bank of India of February 20, 2015.

138.Which of the following is/are correct?

1. USA is India’s top country of export.

2. China is India’s top country of import

Select the Correct Option

A. 1 only

B. 2 only

C. Both are correct

D. None is correct

Answer: C

139.Which of the following is/are correct in context to Taxes in India?

1. In the last decade, Corporate Tax has always been higher than the Income Tax.

2. In the last decade, the Direct Tax to GDP Ratio has always been more than 5%.

Select the Correct Option

A. 1 only

B. 2 only

C. Both 1 & 2

D. None of the above

Answer: C

www.YouTube.com/SleepyClasses www.SleepyClasses.com

89

Jashanjot Sidhu
Jashanjot Sidhu
Jashanjot Sidhu
Jashanjot Sidhu
Jashanjot Sidhu
Jashanjot Sidhu

Explanation

www.YouTube.com/SleepyClasses www.SleepyClasses.com

90

140.The Department of Industrial Policy and Promotion (DIPP) has been renamed to:

A. Department for Industry Advancement

B. Department for Promotion of Industry and Internal Trade

C. Department for New Age Industrial Development

D. None of the above

Answer: B

Explanation

• We discussed it in the last year’s 2-Minute Series.

• The name was changed in the BUDGET of 2019-20.

• It comes under Ministry of Commerce and Industry.

Change Order Given By

• President of India

Ministry earlier responsible for Internal Trade

• Ministry of Consumer Affairs

It will deal with matters related with

• Promotion of internal trade

• Retail trade

• Welfare of traders and their employees

• Facilitating ease of doing business and start-ups

141.Jal Shakti Abhiyan had its primary objective as:

A. Increased power production from Water resources

B. Promoting solar-power based irrigation equipment

C. Promoting a time-bound, mission-mode water conservation campaign

D. None of the above

Answer: C

Explanation

• Focused on Jal Sanchay, the Jal Shakti Abhiyan (JSA) is a time-bound, mission-mode water

conservation campaign.

The JSA will run in two Phases:

• Phase 1 from 1st July to 15th September 2019 for all States and Union Territories.

www.YouTube.com/SleepyClasses www.SleepyClasses.com

91

Jashanjot Sidhu
Jashanjot Sidhu
Jashanjot Sidhu
Jashanjot Sidhu

• Phase 2 from 1st October to 30th November 2019 for States and UTs receiving the retreating

monsoon (Andhra Pradesh, Karnataka, Puducherry and Tamil Nadu).

• During the campaign, officers, groundwater experts and scientists from the Government of India

worked together with state and district officials in India’s most water-stressed districts for water

conservation and water resource management by focusing on accelerated implementation of five

target intervention.

• The JSA aims at making water conservation a Jan Andolan through asset creation and extensive

communication.

142.Which of the following crops can be grown using Ratoon cropping method?

1. Rice

2. Sugarcane

3. Banana

4. Pineapple

Which of the following is/are correct?

A. All except1

B. 1 Only

C. All are Correct

D. None is Correct

Answer: C

Explanation

• Ratoon cropping method, farmers don’t remove the plants, but continue to take a smaller crop on the

same area, which adds to their income.

• Ratooning is the agricultural practice of harvesting a monocot crop by cutting most of the above-

ground portion but leaving the roots and the growing shoot apices intact so as to allow the plants to

recover and produce a fresh crop in the next season.

• This practice is widely used in the cultivation of crops such as rice, sugarcane, banana, and pineapple.

www.YouTube.com/SleepyClasses www.SleepyClasses.com

92

Jashanjot Sidhu
Jashanjot Sidhu
Jashanjot Sidhu
Jashanjot Sidhu

• A Ratoon crop only gives one third or one-fourth of the regular yield.

• Ratoon crops cannot be perennially renewed, and may be harvested only for a few seasons, as a

decline in yield tends to occur due to increased crowding, damage by pests and diseases, and a decline

in soil fertility.

• Recently, it has been found that due to the Ratoon cropping method, pink bollworms are surviving

and thriving in the Vidarbha region.

• Traditionally cotton has been a six-month crop, which is planted in May and June, and harvested by

December, but due to the introduction of BT seeds, the life of a six-month crop has increased to 10 to

12months.

• Due to the Ratoon cropping method, the dormant larvae of pink bollworm survives in the cotton

seeds after harvest, and when the time is right, it again enters the fields.

143.Which of the following is/are correct in context of Renewable Energy (RE) in India?

1. As of December 31, 2019, Wind segment is the leading source of RE in India.

2. Solar energy segment is the key driver of new capacity growth.

Which of the following is/are correct?

A. 1 Only

B. 2 Only

C. Both are Correct

D. None is Correct

Answer: C

Explanation

• As on December 31, 2019, the total grid-connected installed renewable power capacity in India stood

at 85,908 MW.

• Wind segment is still leading now with a total installed capacity of 37,505 MW but Solar is fast

growing and is likely to overtake the wind sector.

• The solar segment’s (which includes ground-mounted and rooftop) total capacity was 33,730 MW as

of December 2019.

• During April-December 2019 period, the renewable energy segment added 7,592 MW of new

capacity when compared with 5,002 MW of new capacity in the year-ago period, an increase of 52

percent.

• The solar energy segment continues to be the key driver of new capacity growth with an addition of

5,013 MW of new capacity through ground-mounted projects and 537 MW of capacity by way of

rooftop projects.

• The wind sector doubled its addition to capacity at 1,879 MW when compared with 993MW.

www.YouTube.com/SleepyClasses www.SleepyClasses.com

93

Jashanjot Sidhu
Jashanjot Sidhu
Jashanjot Sidhu
Jashanjot Sidhu
Jashanjot Sidhu
Jashanjot Sidhu
Jashanjot Sidhu

• The MNRE has fixed a total capacity addition target of 11,802 MW for 2019-20 (15,602 MW

in2018-19).

✓Solar: 8,500 MW (ground-mounted 7,500 MW and rooftop 1,000MW).

✓Wind: 3,000MW

✓Biomass: 250MW

✓Small Hydro: 50MW

144.Which of the following Indian cities find a place in the UNESCO Creative Cities Network?

1. Mumbai

2. New Delhi

3. Jaipur

4. Hyderabad

Which of the following is/are correct?

A. 1, 2 & 3 only

B. 1, 3 & 4 only

C. 2 & 3 only

D. None of the above

Answer: B

Explanation

• UNESCO has designated Mumbai as a member of UNESCO Creative Cities Network (UCCN) in the

field of FILM and Hyderabad in the field of GASTRONOMY.

• UCCN, created in 2004, is a network of cities which are thriving, active centres of cultural activities in

their respective countries. The UNESCO Creative Cities Network now counts a total of 246cities.

• The 7 categories for recognition under UCCN are as follows-

1. Crafts and Folk Arts

2. Design

3. Film

4. Gastronomy

5. Music

6. Media Arts

7. Literature

Previously, 3 Indian cities were recognized as members of UCCN, namely- www.YouTube.com/SleepyClasses

www.SleepyClasses.com 94

• Jaipur-Crafts and Folk Arts (2015)

• Varanasi-Creative city of Music (2015)

• Chennai-Creative city of Music (2017)

145.Dividend Distribution Tax (DDT) is a

A. Direct Tax

B. Indirect Tax

C. Both A & B

D. Neither A nor B

Answer: A

Explanation

• A dividend is a return given by a company to its shareholders out of the profits earned by the

company in a particular year.

• Dividend constitutes income in the hands of the shareholders.

• Before the budget announcement,

• The Dividend Distribution Tax, or DDT, used to be taxable at source, and was deducted at the time of

the company distributing dividends.

✓Dividends will now be taxed in the hands of recipients at their applicable rate.

146.Consider the following:

1. The major ports handle less than 50 per cent of the country’s total cargo traffic.

2. Andhra Pradesh has two major ports.

Which of the above is/are true?

A. 1 only

B. 2 only

C. Both are Correct

D. None is Correct

Answer: D

Explanation

• The major ports handle about 60 per cent of the country’s total cargo traffic.

www.YouTube.com/SleepyClasses www.SleepyClasses.com

95

Jashanjot Sidhu

• There are 12 major ports Deendayal (erstwhile Kandla), Mumbai, JNPT, Mormugao, New Mangalore,

Cochin, Chennai, Kamarajar (earlier Ennore), V.O. Chidambaranar, Visakhapatnam, Paradip and

Kolkata (including Haldia).

• Deendayal port handled the highest traffic volume at 61.04 MT during the April-September period

of2019.

• It is followed by Paradip at 55.55 MT, Visakhapatnam at 34.75, JNPT at 34.41 MT, Kolkata (including

Haldia)at

• 31.64 MT, and Mumbai at 30.10 MT.

• Chennai port handled 24.74 MT of cargo, while New Mangalore handled 17.86 MT.

147.It has been announced in Budget that, Fibre to the Home (FTTH) connections through Bharatnet will

link 100,000 gram panchayats this year.

Consider the following in context of the Optical Fibre used to create the National Optical Fiber

Network (NOFN) in India:

1. Fibre-optic communication is a method of transmitting information from one place to another by

sending pulses of infrared light.

2. Electrical cabling is preferred over Fibre when high bandwidth, long distance, or immunity to

electromagnetic interference are required.

Select the Correct Option www.YouTube.com/SleepyClasses

www.SleepyClasses.com 96

Jashanjot Sidhu
Jashanjot Sidhu

A. 1 Only

B. 2 only

C. Both are Correct

D. None is Correct

Answer: A

Explanation

• Fibre-optic communication is a method of transmitting information from one place to another by

sending pulses of infrared light through an optical fibre.

• The light forms an electromagnetic carrier wave that is modulated to carry information.

• Fibre is preferred over electrical cabling when high bandwidth, long distance, or immunity to

electromagnetic interference are required.

• This type of communication can transmit voice, video, and telemetry through local area networks,

computer networks, or across long distances.

• Optical fibre is used by many telecommunications companies to transmit telephone signals, Internet

communication, and cable television signals.

• The National Optical Fibre Network (NOFN) aims to connect all the 2,50,000 Gram panchayats in the

country and provide 100 Mbps connectivity to all gram panchayats (GPs).

• The NOFN project was funded by the Universal Service Obligation Fund (USOF).

• Based on NOFN experiences, newer, updated and upgraded version – Bharat Net was conceived as a

nation-wide broadband network.

• Bharat Net is a project of national importance to establish, by 2017, a highly scalable network

infrastructure accessible on a non-discriminatory basis, to provide on demand, affordable broadband

connectivity of 2 Mbps to 20 Mbps for all households and on demand capacity to all institutions, to

realise the vision of Digital India, in partnership with States and the private sector.

148.It has been announced in the budget that 5 archaeological sites will be developed as iconic sites with

on-site Museums. Which of the following sites are correctly matched with their respective states?

1. Rakhigarhi Haryana

2. Hastinapur Uttar Pradesh

3. Shivsagar Karnataka

4. Dholavira Rajasthan

5. Adichanallur Kerala

Select the Correct Option

A. 1 Only

www.YouTube.com/SleepyClasses www.SleepyClasses.com

97

Jashanjot Sidhu
Jashanjot Sidhu
Jashanjot Sidhu
Jashanjot Sidhu

B. 1 & 2 Only

C. 4 & 5 only

D. All are Correct

Answer: B

Explanation

These 5 archaeological sites will be developed as iconic sites with on-site Museums.

1. Rakhigarhi – Haryana

2. Hastinapur - Uttar Pradesh

3. Shivsagar – Assam

4. Dholavira – Gujarat

5. Adichanallur - Tamil Nadu

There’s also a proposal to set-up a Maritime museum to be set up at Lothal- the Harrapan age

maritime site near Ahmedabad, by Ministry of Shipping.

149.A sum of Rs. 100 crore has been allocated to begin the preparations for G20 presidency to be hosted

in India. India will host the G20 summit in the year:

A. 2020

B. 2021

C. 2022

D. 2023

Answer: C

Explanation

2022 - which will be India's 75th anniversary as an independent nation.

2020 – Saudi Arabia

2021 - Italy

• The G20 is an international forum for the governments and central bank governors from 19 countries

and the European Union.

• It originated in 1999 at the level of Finance Ministers and Central Bank Governors.

• In the wake of the 2008 global financial crisis, the G20 was elevated to include the leaders of member

countries.

• The first G20 Leaders’ Summit took place in Washington D.C. in November 2008.

www.YouTube.com/SleepyClasses www.SleepyClasses.com

98

Jashanjot Sidhu
Jashanjot Sidhu

• The Group of 20 nations (G20) is an international grouping consisting of: Argentina, Australia, Brazil,

Canada, China, the European Union, France, Germany, India, Indonesia, Italy, Japan, Mexico, Russia,

Saudi Arabia, South Africa, South Korea, Turkey, the UK and the US.

150.It has been announced in the budget that Deposit Insurance and Credit Guarantee Corporation

(DICGC) is permitted to increase Deposit Insurance Coverage to Rs. 5 lakh from Rs.1 lakh per

depositor.

In context of DICGC, consider the following:

1. Regional Rural Banks are exempt from the purview of DICGC.

2. DICGC does not insure deposits of Central/State Governments

Select the Correct Option

A. 1 only

B. 2 only

C. Both are correct

D. None is correct

Answer: B

Explanation

• The following are required to be registered under DICGC act:

1. All commercial banks

2. Regional Rural Banks

3. Co-operative Banks

4. A primary co-operative credit society becoming a primary co-operative bank

• DICGC insures all bank deposits, such as saving, fixed, current, recurring, etc. except the following

types of deposits:

1. Deposits of foreign Governments

2. Deposits of Central/State Governments

3. Inter-bank deposits

4. Deposits of the State Land Development Banks with the State co-operative banks

5. Any amount due on account of and deposit received outside India

6. Any amount which has been specifically exempted by the corporation with the previous approval

of the RBI.

www.YouTube.com/SleepyClasses www.SleepyClasses.com

99

Jashanjot Sidhu
Jashanjot Sidhu
Jashanjot Sidhu
Jashanjot Sidhu
Jashanjot Sidhu
Jashanjot Sidhu

151.Which of the following is/are incorrect Central Drugs Standard Control Organisation (CDSCO)

1. It is under Directorate General of Health Services, Ministry of Health & Family Welfare

2. One of its most important functions is to implement and enforce the provisions of the Drugs Price

Control Order (DPCO)

A. 1 only

B. 2 only

C. Both 1 and 2

D. Neither 1 nor 2

Answer: B

Explanation

• Govt is taking various steps to reduce dependence on import of antibiotic raw materials

• The government has withdrawn exemption of Customs duty on certain categories of bulk drugs/APIs,

which are used in the manufacturing of some the antibiotics, to provide a level-playing field to

domestic manufacturers.

• The Department of Pharmaceuticals has formulated a scheme, namely “Assistance to bulk drug

industry for common facility centre”, for providing assistance to bulk drug segment in any upcoming

bulk drug park promoted by state governments/state corporations

About CDSCO

• The Central Drugs Standard Control Organisation is under Directorate General of Health Services,

Ministry of Health & Family Welfare

• Under the Drugs and Cosmetics Act, CDSCO is responsible for

✓Approval of Drugs

✓Conduct of Clinical Trials

✓Laying down the standards for Drugs

✓Control over the quality of imported Drugs in the country

✓Coordination of the activities of State Drug Control Organisations by providing expert advice with

a view of bring about the uniformity in the enforcement of the Drugs and Cosmetics Act

✓Ensuring the safety, rights and well being of the patients by regulating the drugs and cosmetics

• Further CDSCO along with state regulators, is jointly responsible for grant of licenses of certain

specialised categories of critical Drugs such as blood and blood products, I.V. Fluids and vaccines

NPPA

• National Pharmaceutical Pricing Authority was set up as an attached office of the Department of

Chemicals and Petrochemicals (now Department of Pharmaceuticals since July 2008) on 29th August

1997

www.YouTube.com/SleepyClasses www.SleepyClasses.com

100

• It has been entrusted inter-alia, with the following functions:

✓To implement and enforce the provisions of the Drugs Price Control Order (DPCO), 1995/2013 in

accordance with the powers delegated to it

✓To undertake and/or sponsor relevant studies in respect of pricing of drugs/formulations

✓To monitor the availability of drugs, identify shortages, if any, and to take remedial steps

✓To collect/maintain data on production, exports and imports, market share of individual companies,

profitability of companies etc. for bulk drugs and formulations

✓To deal with all legal matters arising out of the decisions of the Authority

✓To render advice to the Central Government on changes/revisions in the drug policy

✓To render assistance to the Central Government in the parliamentary matters relating to the drug

pricing

152.In news, Shore power, relates with:

A. Total renewable energy potential of a shore

B. Total renewable energy produced on a shore

C. Share of renewable energy in total energy consumed on a shore

D. None of the above

Answer: D

Explanation

• Shore power or shore supply is the provision of shoreside electrical power to a ship at berth while its

main and auxiliary engines are shut down

• Shore-power, also known as cold ironing or alternative maritime power, enables ships at dock or in

dry dock to use shore-side electricity to power on-board electrical systems, such as lighting,

ventilation, communication, cargo pumps and other critical equipment, while turning off their

auxiliary engines

• India’s major ports now run fully on renewable energy

• These ports also offer shore-power to ships to cut costs and emissions

• Shipping Ministry, under the ‘green port’ initiative, directed all the major pots to install grid-

connected and roof top solar and wind power projects to facilitate day-to-day operations including

supplying shore-power to visiting ships in an eco-friendly manner.

153.Vivad se Vishwas Bill, is related with

A. Direct Taxes

B. Indirect Taxes www.YouTube.com/SleepyClasses

www.SleepyClasses.com 101

C. Benami Transactions

D. Promoting Disinvestments

Answer: A

Explanation

• Minister of Finance introduced the Direct Tax Vivad se Vishwas Bill, 2020 on February 5, 2020.

• The Bill provides a mechanism for resolution of pending tax disputes related to income tax and

corporation tax

• The Bill proposes a resolution mechanism under which an appellant can file a declaration to the

designated authority to initiate resolution of pending direct tax disputes.

• The amount payable by the appellant for resolution of disputes is determined based on whether the

dispute relates to payment of tax, or payment of interest, penalty or fee.

• Its scope can be widened to cover litigation pending in various debt recovery tribunals (DRTs).

154.In context of LIC (Life Insurance Corporation of India), consider the following

1. It is a Statutory organisation

2. It started as a private organisation and was nationalised in 1971

A. 1 only

B. 2 only

C. Both 1 and 2

D. Neither 1 nor 2

Answer: A

Explanation

• Life Insurance Corporation of India is an Indian state-owned insurance group and investment

corporation owned by the Government of India.

• LIC was founded in 1956 when the Parliament of India passed the Life Insurance of India Act that

nationalised the insurance industry in India.

• The nationalisation of the life insurance business in India was a result of the Industrial Policy

Resolution of 1956, which had created a policy framework for extending state control over at least 17

sectors of the economy, including life insurance.

155.Which of the following is/are correct in context of Financial Action Task Force (on Money

Laundering):

1. It was founded in 1989 on an OECD initiative.

www.YouTube.com/SleepyClasses www.SleepyClasses.com

102

2. FATF grey lists a country which it considers as a safe haven for terror funding and money

laundering.

3. Pakistan will be the first country ever to be blacklisted.

A. 2 only

B. 1 & 2 only

C. 3 only

D. All are correct

Answer: A

Explanation

• It was founded in 1989 on a G-7 initiative.

• So far, only two countries have been blacklisted, they are Iran and North Korea.

156.Codex Trust Fund is run under the aegis of:

A. WHO

B. FAO

C. Both of the above

D. None of the above

Answer: C

157.Which of the following is/are correct in context of AGR (Adjusted Gross Revenue)?

A. It is a part of profit-sharing model announced in the National Telecom Policy, 1999.

B. It is a part of revenue-sharing model announced in the National Telecom Policy, 1999.

C. It is a part of a fixed annual licence fee announced in the National Telecom Policy, 1999.

D. None of the above

Answer: B

Explanation

• Supreme Court verdict on definition of AGR (Adjusted Gross Revenue) in favour of Department of

Telecommunications (DoT) will blow a Rs 1.48 lakh crore hole in telecos’ balance sheet

• Worst hit are Airtel and Vodafone Idea

www.YouTube.com/SleepyClasses www.SleepyClasses.com

103

www.YouTube.com/SleepyClasses www.SleepyClasses.com

104

158.In context of the Department of Investment and Public Asset Management (DIPAM), which of the

following statements is/are correct

1. It is a part of Ministry of Heavy Industries

2. It was earlier known as the Department of Disinvestment

A. 1 only

B. 2 only

C. Both 1 and 2

D. Neither 1 nor 2

Answer: B

Explanation

• The Department of Disinvestment was set up as a separate department on 10th December 1999 and

was later renamed as Ministry of Disinvestment from 6th September 2001

• From 27th May 2004, the Department of Disinvestment is one of the Departments under the

Ministry of Finance

• The Department of Disinvestment has been renamed as Department of Investment and Public Asset

Management (DIPAM) from 14th April 2016

159.Pandemic bonds were launched by

A. The World Bank

B. World Health Organisation

C. Asian Development Bank

D. None of the above

Answer: A

Explanation

The Ebola outbreak inspired the World Bank to develop a so-called “pandemic catastrophe bond,”.

They were an instrument designed to quickly provide financial support in the event of an outbreak.

What are Catastrophe Bonds?

• They are a way to transfer risk, often for natural disasters.

• If a specific qualifying event occurs, such as if claims from a natural disaster exceed a certain amount,

the bond holders forfeit the principal of the bond, which goes to the insurer to help defray costs.

• Pandemic bonds are triggered by, for example, the number of patients or the speed of disease spread

(a “parametric trigger”).

www.YouTube.com/SleepyClasses www.SleepyClasses.com

105

Jashanjot Sidhu

World Bank’s Pandemic Bonds

• The World Bank announced the creation of the PEF in May 2016 at the G7 Finance Ministers and

Central Governors meeting in Sendai, Japan.

• The PEF will quickly channel funding to countries facing a major disease outbreak with pandemic

potential.

• Its unique financing structure combines funding from the bonds issued today with over-the-counter

derivatives that transfer pandemic outbreak risk to derivative counterparties.

• The bonds are issued by the World Bank's International Bank for Reconstruction and Development

(IBRD).

• The bonds offer investors high coupons in return for the risk of having to forgo some or all their

money in the event of pandemic outbreaks of a number of infectious diseases.

160.In context of ESOP (Employee stock ownership plan) which of the following is/are correct:

1. They help companies to retain employees for a long-term.

2. ESOPs are not yet allowed in India.

A. 1 only

B. 2 only

C. Both are correct

D. None is correct

Answer: A

Explanation

The Finance Minister has assured start-ups to resolve ESOP tax related issue

What are ESOPs

• ESOPs enable employees to buy the company’s shares at a discounted price.

• Companies and, usually, startups roll out this scheme for selected employees, based on their position

and ability to impact the company.

• This ownership comes in the form of company shares, which is assured upon fulfilling predefined

conditions.

• But most often, ESOPs become a part of their compensation offering in startups, to motivate

employees to give their best at work.

• ESOPs give the right to directors, officers and employees of the company to subscribe to the company

share at a future date at a predetermined rate.

• Companies registered as private limited can benefit from this by following the Companies Act, 2013

and rules made thereon.

www.YouTube.com/SleepyClasses www.SleepyClasses.com

106

Jashanjot Sidhu
Jashanjot Sidhu
Jashanjot Sidhu
Jashanjot Sidhu
Jashanjot Sidhu
Jashanjot Sidhu
Jashanjot Sidhu

• Employees who opt to convert the ESOP to shares will have those shares taxable under the Income

Tax Act, 1961.

161.In context of The Essential Commodities Act (ECA) consider the following:

1. The act is applicable on Grains, Pulses and Fertilisers only.

2. The Centre can include new commodities as and when the need arises without waiting for

Parliament’s approval.

Select the Correct Option

A. 1 only

B. 2 only

C. Both are correct

D. None is correct

Answer: B

Explanation

In News

Because of Coronavirus outbreak, Provisions of Essential Commodities Act may be invoked to ensure

adequate supply of drugs.

What is ECA?

• Enacted in 1955.

• Used by the Government to regulate the production, supply and distribution of commodities declared

‘essential’.

• This is done in order to make them available to consumers at fair prices.

What all does it include?

• The list of items under the Act include drugs, fertilisers, pulses and edible oils, and petroleum and

petroleum products.

• The Centre can include new commodities as and when the need arises, and take them off the list once

the situation improves.

• Legal Metrology (Packaged Commodities) Rules 2011 is linked to the ECA.

• The Government can fix the retail price of any packaged commodity that falls under the ECA.

162.Totalizer Machines were in news recently. The Machines are related to:

A. Cricket

B. Artificial Intelligence www.YouTube.com/SleepyClasses

www.SleepyClasses.com 107

Jashanjot Sidhu
Jashanjot Sidhu

C. Elections

D. Machine learning

Answer: C

Explanation

• Simply put, a totaliser is a mechanism which allows votes from 14 booths to be counted together so

that voters are saved from pre-poll intimidation and post-poll harassment.

• Currently, the votes cast via EVMs are counted on individual booth basis. The interface connects to

the main control units of a 14 EVM cluster and the consolidated vote count for candidates is obtained

merely by pressing the result button on the totaliser.

• This result obtained is without disclosure of votes polled by candidates at particular voting booths/

stations.

• Before the introduction of EVMs, ballot papers were often mixed to prevent intimidation of voters by

disclosure of voting pattern.

163.Which of the following is/are true about FOREX?

1. Only US Dollars are considered as Foreign Currency Assets (FCAs)

2. In terms of assets with IMF, only Special Drawing Rights are considered (and not RBI’s reserve

position)

Select the Correct Option

A. 1 only

B. 2 only

C. Both are correct

D. None is correct

Answer: D

Explanation

• Foreign currency assets are considered but their value is denominated in US dollar terms

www.YouTube.com/SleepyClasses www.SleepyClasses.com

108

164.Which of the following is correct in context of investment in Sovereign Gold Bonds?

1. They are issued by Reserve Bank of India.

2. The minimum investment in these bonds is one gram

A. 1 only

B. 2 only

C. Both are correct

D. None of the above

Answer: C

Explanation

What is Sovereign Gold Bond (SGB)?

• SGBs are government securities denominated in grams of gold. They are substitutes for holding

physical gold.

Who is the issuer?

• The Bond is issued by Reserve Bank on behalf of Government of India.

What is the minimum and maximum limit for investment?

• The Bonds are issued in denominations of one gram of gold and in multiples thereof.

• Minimum investment in the Bond shall be one gram with a maximum limit of subscription of 4 kg for

individuals.

What will I get on redemption?

• On maturity, the redemption price shall be based on simple average of closing price of gold of 999

purity of previous 3 business days from the date of repayment, published by the India Bullion and

Jewellers Association Limited

165.In context of SAARC Currency Swap Facility, which of the following is/are correct:

1. SAARC Currency Swap Facility came into operation in 2012.

2. The present swap arrangement is of overall corpus of US $10 billion.

3. The drawals can be made in US Dollar, Euro or Indian Rupee.

A. 1 & 2 only

B. 2 & 3 only

C. 1 & 3 only

D. All of the above

Answer: C

www.YouTube.com/SleepyClasses www.SleepyClasses.com

109

Explanation

SAARC Currency Swap Facility 

• To further financial stability and economic cooperation within the SAARC region, the Reserve Bank of

India, with the concurrence of the Government of India, has decided to put in place a revised

Framework on Currency Swap Arrangement for SAARC countries 2019-2022.

• The Framework is valid from November 14, 2019 to November 13, 2022.

• Based on the terms and conditions of the Framework, the RBI would enter into bilateral swap

agreements with SAARC central banks, who want to avail swap facility.

• It may be recalled that the SAARC Currency Swap Facility came into operation on November 15,

2012 with an intention to provide a backstop line of funding for short term foreign exchange liquidity

requirements or balance of payment crises till longer term arrangements are made.

• Under the Framework for 2019-22, RBI will continue to offer swap arrangement within the overall

corpus of US $ Two billion.

• The drawals can be made in US Dollar, Euro or Indian Rupee.

• The Framework provides certain concessions for swap drawals in Indian Rupee.

• The Currency Swap Facility will be available to all SAARC member countries, subject to their signing

the bilateral swap agreements.

166. Which of the following is/are true about PFMS (Public Financial Management System)?

1. It was established in 2016 after the demonetization drive, to effectively track the amount of

money deposited in public banks by people.

2. It is also enabling the payment through Direct Benefit Transfer (DBT) to beneficiaries.

A. 1 only

B. 2 only

C. Both are correct

D. None is correct

Answer: B

Explanation

• The Public Financial Management System (PFMS), earlier known as Central Plan Schemes Monitoring

System (CPSMS), is a web-based online software application developed and implemented by the

Office of Controller General of Accounts (CGA).

• PFMS was initially started during 2009 as a Central Sector Scheme of Planning Commission with the

objective of tracking funds released under all Plan schemes of GoI, and real time reporting of

expenditure at all levels of Programme implementation.

www.YouTube.com/SleepyClasses www.SleepyClasses.com

110

Jashanjot Sidhu
Jashanjot Sidhu
Jashanjot Sidhu

• Subsequently in the year 2013, the scope was enlarged to cover direct payment to beneficiaries

under both Plan and non-Plan Schemes.

• The latest enhancement in the functionalities of PFMS commenced in late 2014, wherein it has been

envisaged that digitization of accounts shall be achieved through PFMS and the additional

functionalities would be built into PFMS in different stages.

• The primary objective of PFMS is to facilitate sound Public Financial Management System for

Government of India (GoI) by establishing an efficient fund flow system as well as a payment cum

accounting network.

• PFMS provides various stakeholders with a real time, reliable and meaningful management

information system and an effective decision support system, as part of the Digital India initiative of

GoI.

The Public Financial Management System (PFMS)

It is also taking care of payments made in:

• DBT

• Centrally Sponsored Scheme (CSS)

• Central Sector (CS) Scheme

• MNREGA

167.In context of Hydroponics, which of the following is/are correct?

1. Here, plants are grown without the use of soil.

2. It is also referred to as aquaculture.

A. 1 only

B. 2 only

C. Both are correct

D. None of the above

Answer: A

Explanation

• A US farmers’ organization has filed a lawsuit claiming that the food grown without mud is not organic

• Hydroponics is a branch of agriculture where plants are grown without the use of soil.

• The nutrients that the plants normally derive from the soil are simply dissolved into water instead,

and depending on the type of hydroponic system used, the plant's roots are suspended in, flooded

with or misted with the nutrient solution so that the plant can derive the elements it needs for

growth.

• It can sometimes be mistakenly referred to as aquaculture.

www.YouTube.com/SleepyClasses www.SleepyClasses.com

111

Jashanjot Sidhu
Jashanjot Sidhu
Jashanjot Sidhu
Jashanjot Sidhu

• Aquaculture is the farming and husbandry of freshwater and marine animals and plants in controlled

environments.

168.Yellow Revolution is related with increasing

A. The production of Oilseeds

B. The production of Cow milk

C. The production of Turmeric

D. The penetration of Gold Bonds

Answer: A

Explanation

• Yellow revolution will attempt to boost oilseeds cultivation in the country to meet the growing edible

oils demand.

• The average vegetable oil seed production in India is 0.3 tonnes per hectare per year compared to the

Asian average of 1.2 tonnes per hectare per year.

169.Which of the following is/are true

1. In terms of Nominal GDP, India has risen to become the world’s fifth largest economy.

2. If prices change from one period to the next and the output does not change, the nominal GDP

would change even though the output remained constant.

A. 1 only

B. 2 only

C. Both are true

D. Neither is true

Answer: C

Explanation

According to data from the IMF’s October 2019 World Economic Outlook,

• India has risen to become the world’s fifth largest economy, when ranked by nominal GDP.

• India, in this ranking, has leapfrogged France and the UK.

Real GDP

• The real GDP is the total value of all of the final goods and services that an economy produces during

a given year, accounting for inflation.

• It is calculated using the prices of a selected base year.

www.YouTube.com/SleepyClasses www.SleepyClasses.com

112

Jashanjot Sidhu
Jashanjot Sidhu

• To calculate Real GDP, it is determined how much GDP has changed because of inflation since the

base year. Real GDP, therefore, accounts for the fact that if prices change but output doesn’t, Real

GDP would not change.

Nominal GDP

• It is calculated by using the prices that are current in the year in which the output is produced. A

nominal value can change due to shifts in quantity and price. If prices change from one period to the

next and the output does not change, the nominal GDP would change even though the output

remained constant.

170.Pharmexcil falls under the aegis of:

A. Department of Pharmaceuticals (under the Ministry of Chemicals and Fertilizers)

B. Ministry of Health and Family Welfare

C. Ministry of Commerce and Industry

D. Ministry of Ayush

Answer: C

171.Which of the following is/are true about Additional Tier-1 (AT1) bonds?

1. RBI is planning to introduce them in later part of 2020.

2. They pay a fixed rate of interest at regular intervals.

A. 1 only

B. 2 only

C. Both are correct

D. None is correct

Answer: B

Explanation

• The RBI’s decision to extinguish additional tier-1 (AT1) bonds of Yes Bank.

• It will hurt a large number of retail and other investors (through write-offs in mutual funds and

insurance funds), bank treasuries and high net worth individuals (HNIs).

• These are also called Innovative Perpetual Debt Instruments (IPDI).

• They also form part of the tier-1 capital of a bank.

What are AT1 bonds?

• AT1 (Additional tier-1) bonds are a kind of perpetual bonds without any expiry date that banks are

allowed to issue to meet their long-term capital requirement.

www.YouTube.com/SleepyClasses www.SleepyClasses.com

113

Jashanjot Sidhu
Jashanjot Sidhu
Jashanjot Sidhu
Jashanjot Sidhu
Jashanjot Sidhu

• That’s why these bonds are treated as quasi-equity instruments under the law.

• The RBI is the regulator for these bonds.

• These bonds are like equity as those who invest in these do not have the same rights as bondholders.

• In case of liquidation, these bondholders are paid only after meeting the obligations to deposit

holders and other bond holders.

Do these bonds pay interest?

• Yes.

• AT1 bonds are like any other bonds issued by banks and companies, which pay a fixed rate of interest

at regular intervals.

• Usually, these bonds pay a slightly higher rate of interest compared to similar, non-perpetual bonds.

• However, the issuing bank has no obligation to pay back the principal to investors.

Are these bonds traded in the market?

• Yes.

• These bonds are listed and traded on the exchanges. So, if an AT1 bondholder needs money, he can

sell it in the market.

How are AT1 bonds redeemed?

• Investors cannot return these bonds to the issuing bank and get the money.

• This means there is no put option available to its holders.

• However, the issuing banks have the option to recall AT1 bonds issued by them (termed call option).

• They can go for a call option five years after these are issued and then every year at a pre-announced

period.

• This way, the issuing banks can give an exit option to AT1 bondholders.

At present, what’s the total value of AT1 bonds in the market?

• According to a report by rating agency ICRA, nearly Rs 94,000 crore worth of AT1 bonds are

currently issued by various banks.

• Of this, Rs 55,000 crore is from PSU banks, while the balance Rs 39,000 crore is from private lenders.

172.In context of Foreign Currency Convertible Bonds (FCCBs), which of the following is/are correct?

1. FCCBs are bonds issued by listed companies in the overseas market.

2. FCCBs usually carry clauses which allow the issuer or bondholder the option to convert the bonds

into shares mid-way during their term.

A. 1 only

B. 2 only www.YouTube.com/SleepyClasses

www.SleepyClasses.com 114

Jashanjot Sidhu
Jashanjot Sidhu
Jashanjot Sidhu
Jashanjot Sidhu
Jashanjot Sidhu
Jashanjot Sidhu
Jashanjot Sidhu
Jashanjot Sidhu
Jashanjot Sidhu
Jashanjot Sidhu
Jashanjot Sidhu
Jashanjot Sidhu
Jashanjot Sidhu
Jashanjot Sidhu
Jashanjot Sidhu
Jashanjot Sidhu
Jashanjot Sidhu
Jashanjot Sidhu
Jashanjot Sidhu
Jashanjot Sidhu
Jashanjot Sidhu

C. Both are correct

D. None of the above

Answer: C

Explanation

Why in News?

• Bharti Airtel raised a total of $3 billion through foreign currency convertible bonds (FCCBs) under the

Reserve Bank of India’s (RBI) approval route.

• This is done primarily to meet its statutory obligations on adjusted gross revenues (AGR) payable to

the telecom department.

What are FCCBs?

• FCCBs are bonds issued by listed companies in the overseas market.

• FCCBs usually carry clauses which allow the issuer or bondholder the option to convert the bonds

into shares mid-way during its term, at a pre-agreed price.

• If the market is bullish in nature, the conversion price is set high.

• Often, with global interest rates far lower than those in India, domestic companies find it cheaper to

raise money through these bonds.

173.The state which has the maximum installations of solar rooftop plants across the country is:

A. Rajasthan

B. Maharashtra

C. Gujarat

D. Tamil Nadu

Answer: C

Explanation

• Of 79,950 systems installed across the country, Gujarat has topped with 64 per cent or two-thirds of

total domestic solar rooftop installations.

• Gujarat has topped the list in the installations of solar rooftop plants across the country with about

50,915 systems being fixed on domestic rooftops in the State as on March 2, 2020.

• Maharashtra follows with 5,513 installations as on the same date.

• The State government has adopted a solar rooftop scheme — Surya Gujarat — to cover about eight

lakh domestic electricity consumers under the scheme by 2022.

www.YouTube.com/SleepyClasses www.SleepyClasses.com

115

Jashanjot Sidhu
Jashanjot Sidhu
Jashanjot Sidhu
Jashanjot Sidhu
Jashanjot Sidhu
Jashanjot Sidhu
Jashanjot Sidhu
Jashanjot Sidhu

174.Select the correct out of the following statements:

1. NDP = GDP + Depreciation

2. NNP = GNP – Depreciation

3. NNP = GDP + Income from abroad – Depreciation

4. GDP = NDP – Depreciation

A. 1 & 2 Only

B. 2 & 3 Only

C. 1, 3 & 4 Only

D. All of the Above

Answer: B

Explanation

• NDP is the GDP calculated after adjusting the weight of the value of depreciation.

✓Thus, a net form of GDP.

✓Therefore, GDP = GDP - Depreciation

• GNP is the GDP of a country added with its “income from abroad”

✓GNP = GDP+ Income from abroad

• NNP on the other hand is the GNP after deducting the loss due to depreciation.

✓Thus, NNP = GNP - Depreciation or

✓NNP = GDP+ Income from abroad - Depreciation

175.Which of the following is/are true in context of Gender Gap Report:

1. It is published by World Economic Forum.

2. It has 5 key dimensions.

3. It divides the world in eight broad geographical groupings.

4. For all sub-indices, the highest possible score is 1.

5. Since 2006, India has consistently been improving its rank.

A. All are correct except 2 & 5

B. Only 2 & 5 are correct

C. All are correct except 4

D. All are correct

Answer: A www.YouTube.com/SleepyClasses

www.SleepyClasses.com 116

Jashanjot Sidhu

Explanation

• It has 4 key dimensions.

• India’s score has consistently dropped.

176.Consider the following regarding World Bank

1. IDA – It focuses mainly on the private sector.

2. IBRD – It provides debt financing to governments that are considered middle income.

3. IFC - A group that gives interest-free loans to the governments of poor countries.

Which of the above is/are correctly matched?

A. 1 and 3 only

B. 1 and 2 only

C. 2 only

D. All of the above

Answer: C

Explanation

• The World Bank has expanded from the single institution that was created in 1944 to a group of five

unique and cooperative institutional organizations.

• The first organization is the International Bank for Reconstruction and Development (IBRD), an

institution that provides debt financing to governments that are considered middle income.

• The second organization within The World Bank is the International Development Association (IDA),

a group that gives interest free loans to the governments of poor countries. 

• The International Finance Corporation (IFC), the third organization, focuses on the private sector and

provides developing countries with investment financing and financial advisory services. 

• The fourth part of The World Bank is the Multilateral Investment Guarantee Agency (MIGA), an

organization that promotes foreign direct investments in developing countries. 

• The fifth and final organization is the International Centre for Settlement of Investment Disputes

(ICSID), an entity that provides arbitration on international investment disputes.

177.Information Fusion Centre – Indian Ocean Region (IFC-IOR) is:

A. A high speed internet platform, operated using Indian Satellites for IOR countries, built by India.

B. A center dedicated to comprehensive maritime domain awareness and share information on vessels

of interest.

C. A state of the art high temperature Fusion Reactor being built by India for and with IOR countries.

www.YouTube.com/SleepyClasses www.SleepyClasses.com

117

Jashanjot Sidhu
Jashanjot Sidhu
Jashanjot Sidhu
Jashanjot Sidhu
Jashanjot Sidhu
Jashanjot Sidhu
Jashanjot Sidhu

D. None of the above

Answer: B

Explanation

• IFC-IOR is established with the vision of strengthening maritime security in the region and beyond, by

building a common coherent maritime situation picture and acting as a maritime information hub for

the region.

• Establishment of IFC- IOR would ensure that the entire region is benefited by mutual collaboration

and exchange of information and understanding the concerns and threats which are prevalent in the

region.

• In December 2018, IFC-IOR centre marked a beginning of a new era wherein collaborative, inclusive

and high tech approach would be adopted to detect and deter maritime security threats of the region.

• The IFC has been established at Gurugram, India and is collocated with Information Management and

Analysis Centre which is jointly administered by the Indian Navy and Indian Coast Guard.

How will it work

• The information Exchange at the IFCIOR would be initially undertaken by virtual means, using

telephone calls, faxes, emails and video conferencing over internet.

• Subsequently, to enable better interconnection, quicker analysis of information and provide timely

inputs, the IFC-IOR would host Liaison Officers from partner countries.

• Additionally, towards enhancing capability building, the IFC-IOR would undertake conduct of

exercises and training capsules in maritime information collection and sharing.

• The setting up of IFC-IOR underscores the governmental approach and effort in line with the vision of

Prime Minister towards Security and Growth of All in the Region (SAGAR).

178.The Central Government compensates its employees for the rise in cost of living by raising the

dearness allowance (DA) it pays them. The DA is calculated based on the:

A. All-India Wholesale Price Index

B. All-India Consumer Price Index

C. All-India Producer Price Index

D. None of the Above

Answer: B

Explanation

• As DA is provided to employees to protect against the price rise in a particular financial year, it is

calculated twice every year – in January and July.

• In the middle of a Pay Commission cycle (every 10 years), the Centre compensates its employees for

the rise in cost of living by raising the dearness allowance (DA) it pays them.

www.YouTube.com/SleepyClasses www.SleepyClasses.com

118

Jashanjot Sidhu
Jashanjot Sidhu
Jashanjot Sidhu
Jashanjot Sidhu

• The formula to calculate the dearness allowance was changed in 2006 by the Government.

• The DA is calculated based on a percentage of the basic salary and is part of the other allowances that

a government or a government-owned company’s employee gets. The DA is calculated based on the

All-India Consumer Price Index (AICPI) for the past 12 months.

• DA is fully taxable for salaried employees.

179.Recycle and re-use is the mantra for eco-friendly living. In this light, FSSAI is implementing RUCO.

RUCO stands for:

A. Repurpose Used Cooking Oil

B. Reducing Used Cooking Oil

C. Reduction of Used Cooking Oil

D. Redefining Usage of Cooking Oil

Answer: A

Explanation

• Food Safety and Standards Authority of India (FSSAI) in association with the Biodiesel Association of

India (BDAI) has recently flagged off the ‘RUCO’ project, short for repurpose used cooking oil.

• RUCO is a project that plans to convert vegetable oils, animal fats or restaurant grease that has

already been used in cooking into biodiesel for running diesel vehicles, or indeed any equipment that

uses diesel.

• Also being done in countries such as the UK and the US

• FSSAI has released a detailed guidance note on the handling and disposal of used cooking oil (UCO) by

small and big food business operators and household-level users.

• RUCO enables the collection of inedible cooking oil by authorised agencies from institutional and

individual users, which is then transferred to a plant to make bio-diesel.

• This fuel is then proposed to be blended with vehicle fuel produced by oil marketing companies for

distribution at the pump.

• FSSAI has notified the limit of total polar compounds (TPC) used in cooking at 25 per cent.

• TPC is used to measure the quality of oil, and its level increases every time oil is re-heated.

• TPC beyond 25 per cent is considered unfit for human consumption.

• Government hopes to address two other issues through RUCO.

• One, preventing adulteration of new cooking oil with UCO in the market;

• Two, reducing repeated usage of the same UCO by food joints keen to cut corners.

www.YouTube.com/SleepyClasses www.SleepyClasses.com

119

Jashanjot Sidhu
Jashanjot Sidhu
Jashanjot Sidhu
Jashanjot Sidhu
Jashanjot Sidhu
Jashanjot Sidhu
Jashanjot Sidhu
Jashanjot Sidhu

• The FSSAI’s new guidelines require large food business operators to store UCO separately, which

they can then sell to authorised UCO aggregators or collection agencies registered with the BDAI,

State biodiesel boards and other agencies nominated by State government.

• Presently, there are 11 authorised aggregators in the country.

• The FSSAI also requires all food business operators to refrain from buying UCO and to monitor the

quality of oil that they use for cooking.

180.Which of the following is/are true in context of T-Bills (Treasury-Bills):

1. They are issued in 2 tenors.

2. They pay quarterly instalments of interest to the holders.

A. 1 only

B. 2 only

C. Both are correct

D. None is correct

Answer: D

Explanation

• Treasury-Bills are government borrowings for less than one year.

• Issued in three tenors:

✓91

✓182

✓364-day

• T-Bills are zero coupon instruments.

• They are issued at a discount and redeemed at face value

181.Consider the following

1. Petroleum and Natural Gas Regulatory Board (PNGRB) is the first regulatory body set up by the

Government of India.

2. One of the tasks of PNGRB is to ensure competitive markets for gas.

3. Appeals against the decisions of PNGRB go before the Appellate Tribunals for Electricity.

Which of the statements given above are correct?

A. 1 and 2 only

B. 2 and 3 only

www.YouTube.com/SleepyClasses www.SleepyClasses.com

120

Jashanjot Sidhu
Jashanjot Sidhu

C. 1 and 3 only

D. 1, 2 and 3

Answer: B

Explanation

• It was formed in 2006

182.In the context of any country, which one of the following would be considered as part of its social capital?

A. The proportion of literate in the population

B. The stock of its buildings, other infrastructure and machines

C. The size of population in the working age group

D. The level of mutual trust and harmony in the society

Answer: D

183.Which of the following is issued by registered foreign portfolio investors to overseas investors who want to be part of the Indian stock market without registering themselves directly?

A. Certificate of Deposit

B. Commercial Paper

C. Promissory Note

D. Participatory Note

Answer: D

184.Which of the following is not the most likely measure the Government/RBI takes to stop the slide of India rupee?

A. Curbing imports of non-essential goods and promoting exports

B. Encouraging Indian borrowers to issue rupee denominated Masala Bonds

C. Easing conditions relating to external commercial borrowing

D. Following an expansionary monetary policy

Answer: D

185.Consider the following statements :

1. According to the Indian Patents Act, a biological process to create a seed can be patented in India.

2. In India, there is no Intellectual Property Appellate Board.

www.YouTube.com/SleepyClasses www.SleepyClasses.com

121

Jashanjot Sidhu

3. Plant varieties are not eligible to be patented in India.

Which of the statements given above is/are correct?

A. 1 and 3 only

B. 2 and 3 only

C. 3 only

D. 1, 2 and 3

Answer: C

www.YouTube.com/SleepyClasses www.SleepyClasses.com

122

Jashanjot Sidhu